Vous êtes sur la page 1sur 108

Enunciado Auxiliar No1 FI2A2

Prof. Aux.: Felipe L. Benavides


Fecha: Lunes 4 de Agosto de 2008

I. Problemas
Problema 1

Consideremos una cinta infinita de densidad superficial σ y ancho W . En algún punto de la cinta existe un agujero
circular de radio R, con 2R < W . Calcular el campo eléctrico en el eje de simetrı́a del agujero mencionado.

Figura No 1.

Indicación: Considere conocido el campo eléctrico producido por un disco de radio conocido y densidad de carga
superficial constante.

Problema 2

Dada una fuente cargada consistente en:

1. Un plano infinito (Π), cargado con densidad de carga uniforme σ.


2. Una recta infinita, (L1 ), cargada con densidad uniforme λ, que forma un ángulo agudo 2α con el plano.

Calcular el campo eléctrico total en un punto P sobre una segunda recta, (L2 ), que bisecta el ángulo entre la recta
y el plano. El punto P está a altura h sobre el plano.

Figura No 2.

1
Problema 3

En el interior de una esfera de radio a y centrada en el origen, se tiene una distribución de carga dada por
5k 2
ρ(r ≤ a) = r , ρ(r > a) = 0
4πa5
Obtener:

1. Carga total de la esfera.


2. Campo eléctrico en todas partes.
3. Potencial eléctrico en todas partes.

4. Divergencia del campo eléctrico dentro y fuera de la esfera.

Problema 4

Un cilindro infinito de radio a tiene densidad de carga volumétrica desconocida aunque se sabe que ella tiene simetrı́a
de rotación con respecto al eje del sistema. El cilindro está rodeado de espacio vacı́o. Se sabe que el campo eléctrico
en el interior del cilindro, expresado en coordenadas cilı́ndricas, está dado por:


− A 2
E = ρ ρ̂
3ε0
donde A es una constante conocida.
Determine:

1. Densidad de carga en todo el espacio.


2. Campo eléctrico fuera del cilindro.
3. Potencial V en todas partes y tal que se cumpla que V (ρ = a) = 0.

2
Resolución Auxiliar No1 FI2A2
Prof. Auxiliar: Felipe L. Benavides
Fecha: Lunes 4 de Agosto, 2008.

I. Problemas
Problema 1

Dada la complejidad geométrica del problema, facilitaremos el cálculo con el ppio. de superposición, obteniendo
el valor deseado como resta entre los valores de campo eléctrico para la cinta completamente cargada, y un disco,
también totalmente cargado. Para éste último, consideraremos el resultado obtenido en el apunte y visto en cátedra,
que determina que dicho campo eléctrico es
 

− σ z z
E = k̂ −√
2ε0 kzk z 2 + R2
con R el radio del disco. Ası́, sólo necesitamos obtener el primer campo eléctrico y restar.

Por definición:


− → 1
Z →
−r −→
−r0
E (−
r = z k̂) = →
− →
− dq(→

r 0)
4πε0 k r − r 0 k3

Al parametrizar, tendremos que: →



r = z k̂; →

r 0 = x0 î + y 0 ĵ; dq(→

r 0 ) = σdx0 dy 0 con− w2 ≤ x ≤ w
2, y −∞ ≤y≤ ∞

Entonces
w
+∞
(z k̂ − x0 î − y 0 ĵ)σdx0 dy 0
Z Z

− 1 2
E = 3
4πε0 −∞ −w
2
(z 2 + x02 + y 02 ) 2

De los tres términos generados, por simetrı́a, sólo el primero es no nulo, pues se trata en los dos casos siguientes
de integrales de funciones impares en intervalo simétrico. El cálculo se reduce a:
w w 3
+∞ +∞
dx0 dy 0 dx0 dy 0 (z 2 + x02 ) 2
Z Z Z Z

− zσ k̂ 2 zσ k̂ 2
E = 3 = 3 3
4πε0 −∞ −w
2
(z 2 + x02 + y 02 ) 2 4πε0 −w
2
(z 2 + x02 ) 2 −∞ (z 2 + x02 + y 02 ) 2

Usando
√ el conveniente cambio:
x 02 + z 2 tg(u) = y 0

x02 + z 2 sec2 (u)du = dy 0
w π √ 3
dx0 x02 + z 2 sec2 (u)du(z 2 + x02 ) 2
Z Z
zσ k̂ 2 2
= 3 3
4πε0 −w
2
(z 2 + x02 ) 2 −π
2 ((z 2 + x02 )sec2 (u)) 2

w π w
dx0 dx0
Z Z Z
zσ k̂ 2 2 zσ k̂ 2
= cos(u)du =
4πε0 −w
2
(z + x02 )
2
−π
2
2πε0 −w
2
(z 2 + x02 )

Por último, con:


ztg(α) = x0
zsec2 (α)dα = dx0

1
arctg ( 2z
w
)
zsec2 (α)dα
Z
zσ k̂ σ k̂ h w  w i σ k̂ w
= 2 2
= arctg − arctg − = arctg
2πε0 arctg (− 2z
w
) z sec (α) 2πε0 2z 2z πε0 2z

Con el resultado obtenido, sólo basta plantear la resta.

Problema 2

Dada la naturaleza de la geometrı́a de la situación, se hace necesario utilizar el ppio. de superposición. Eso sı́, en
éste caso podemos ocupar adicionalmente la Ley de Gauss, facilitando la obtención del campo eléctrico. La idea
es primero utilizar lo anterior y encontrar con ello el valor de los campos en P, para luego sumarlos. (Para lo que
habrá que efectuar una transformación entre los sistemas de referencia).

Para la recta, utilizamos un cilindro que encierre un cierto volúmen, con dicha recta en su eje. Denotamos A la
altura de él, información sólo necesaria para caracterizarlo, prescindible en realidad. Después, notamos posible
dirección y/o coordenada(s) de dependencia del campo, y con todo lo anterior se lleva a cabo el cálculo. Dada la
simetrı́a del problema, (en altura y orientación se enfrenta el mismo escenario), es razonable suponer que el campo
depende del radio, y que su dirección apunta según rho.
Z
− −
→ → QV
E · dS =
S=dV ε0



E = E(ρ)ρ̂

Si bien la superficie S representa toda la cubierta del volúmen del cilindro, sólo ciertas partes son de interés. Las
tapas, arriba y abajo, tienen normales perpendiculares a ρ, lo que muestra que los productos puntos en la integral
serán cero. De ésta forma, sólo importa el manto del cilindro. Ası́:
Z A Z 2π
Aλ →
− λ
E(ρ)ρ̂·ρ̂ρdθdz = ⇒ E (ρ) = ρ̂
0 0 ε 0 2πε 0ρ

Para el plano, efectuamos un razonamiento análogo. Por conveniencia, nuevamente elegimos un cilindro, que
será quien encierre, eventualmente, toda la carga. Esta vez eso sı́, la simetrı́a del problema cambia, por lo que las
suposiciones sobre dependencia y dirección del campo son igualmente distintas.
Sabemos que hay simetrı́a en orientación y radio, pero no necesariamente en altura. Por ello, podemos asumir
que el campo dependerá, en el peor caso, de z. Por la forma del problema, se sabe con certeza que la dirección
del campo es según k̂, o -k̂, dependiendo del sistema de referencia y la posición en el espacio. Para ésta situación
podemos asumir k̂, colocando correspondientemente el sist. de referencia. Adicionalmente vemos que sólo las tapas
del cilindro son de interés, pues el manto, con normal según ρ̂, anula su aporte a la integral. Ası́, (ojo que el aporte
de las tapas es doble):
Z 2π Z ρ
σπρ2 →
− ρ2 σπρ2 →
− σ
E(z)k̂·k̂ρdρdθ = ⇒ 2 E (z) 2π = ⇒E = k̂
0 0 ε 0 2 ε 0 2ε0

Para unir los resultados basta intercambiar ρ por h en el campo producido por la recta, (para referir al punto
P directamente, ya que el aporte del campo generado por el plano es cte. independiente de la posición), y escribir
ρ̂ en el sistema de referencia del plano. De un sencillo análisis geométrico,

cos(90 − 2α)î − sen(90 − 2α)k̂ = ρ̂ ⇒


− σ λ  
E (P ) = k̂ + cos(90 − 2α)î − sen(90 − 2α)k̂
2ε0 2πε0 h
Por supuesto, queda a criterio de quien resuelva el problema el sistema en que quiera escribir la solución, mientras
sea ortonormal y consistente en todo el término.

2
Problema 3

(1) Planteando la integral, tendremos que:


Z Z

− 5k 2 2
ρ( r)dV = ρ ρ sen(θ)dρdθdϕ = Qtotal ⇒
esf era esf era 4πa5

2π π a
a5
Z Z Z  
5k 4 5k
ρ sen(θ)dρdθdϕ = (2) (2π) = k
0 0 0 4πa5 4πa5 5

(2) Nuevamente por Ley de Gauss, asumiendo:




E = E(ρ)ρ̂
Dada la distribución de cargas, finita, debemos separar los casos para el campo eléctrico en ρ. Usando coordenadas
esféricas, se tiene:

Si ρ ≤ a
2π π 5k 4
R
4πa5 ρ sen(θ)dρdθdϕ
Z Z
2
E(ρ)ρ̂ · ρ̂ρ sen(θ)dθdϕ = ⇒
0 0 ε0

5k ρ5 →
− kρ3
E(ρ)ρ2 4π = 5
4π ⇒ E (ρ) = ρ̂
4πa 5ε0 4πa5 ε0

Si ρ > a
Z

− → Qtotal
− k
E (ρ)dS = = ⇒
ε0 ε0

Z 2π Z π
k →
− k
E(ρ)ρ̂ · ρ̂ρ2 sen(θ)dθdϕ = ⇒ E (ρ) = ρ̂
0 0 ε0 4πρ2 ε0
Observación: Es importante notar que el campo es contı́nuo en a. Esto no es algo trivial y en general debiera
analizarse por separado.

(3) El potencial eléctrico corresponde al valor del trabajo que se necesita para alejar o acercar una carga hacia otra
disposición de ellas. Como depende directamente del campo eléctrico, (dada su definición), el cálculo en ésta parte
también estará diferenciado según el radio de la esfera fuente. Dado que ésta posee una cantidad finita de cargas,
se asume que V(ρ = ∞) = 0. Por definición:
Z →−
r

− − →
→ −
V(r)=− E · dr ⇒


r0
Si ρ > a
Z ρ
k k
V (ρ) = − ρ̂· − dρρ̂ = −
∞ 4πε0 ρ2 4πρε0
Observación: Notar que en el resultado anterior la dirección del diferencial de posición es en contra del vector
unitario ρ̂, pues la partı́cula cargada es traı́da desde el infinito hasta una posición a distancia radial ρ de la esfera.
Si ρ ≤ a
Z a Z ρ
kρ3
 4
a4

k k k ρ
V (ρ) = − 2
ρ̂· − dρρ̂ + − 5
ρ̂· − dρρ̂ = − + −
∞ 4πε0 ρ a 4πa ε0 4πaε0 4πε0 a5 4 4

3
(4) La Ley de Gauss posee una forma en la que se expresa como sigue:


− ρ(→

r)
∇· E =
ε0
Con ésto, es fácil ver que lo pedido corresponde a los datos entregados. La divergencia del campo eléctrico es
nula fuera de la esfera, y vale la densidad de carga volumétrica dada al interior, dividida por la cte. ε0 .

Problema 4

Ppto.
Se sugiere comenzar con la fórmula que entrega la Ley de Gauss para situaciones con distribuciones volumétricas,
recién citada.

4
Enunciado Auxiliar No2 FI2A2
Prof. Aux: Felipe L. Benavides
Fecha: Lunes 11 de Agosto

Problema 1

Suponga que dos cargas puntuales en el vacı́o,−q y + 12 q, se sitúan en el origen y en el punto (a, 0, 0), respectivamente.
(Usando coordenadas cartesianas)

a) ¿En qué punto del eje x se anula el campo eléctrico?

b) Demuestre que la superficie equipotencial ϕ = 0 tiene forma esférica. ¿Cuáles son las coordenadas del centro de
dicha esfera?
Problema 2
Situaciones Aplicadas
V 
a) El campo eléctrico en la atmósfera sobre la superficie terrestre es aproximadamente 200 m , dirigido hacia abajo.
V 
A 1400 [m] por encima de la superficie terrestre, el campo eléctrico de la atmósfera es de sólo 20 m , dirigido hacia
abajo también. ¿Cuál es la densidad media de carga en la atmósfera por debajo de los 1400 [m]?

b) La resistencia
 V  dieléctrica del aire, (es decir, el campo eléctrico lı́mite a partir del cual el aire llega a ser conductor)
es de 3 · 106 m . ¿Cuál es el máximo potencial posible de un conductor esférico aislado, de 10 cm. de radio, en el
aire? (Potencial tal que no sobrepase la resistencia dieléctrica del medio). ¿Cuál es el radio que debiera tener un
conductor esférico que, en las mismas condiciones anteriores de potencial máximo en el aire sin descarga, tuviese
una carga de 1 coulomb?
Problema 3

Considere una esfera de radio R en el vacı́o, con sólo un hemisferio, (casquete o mitad), cargado, con densidad
de carga superficial constante σ. Calcule el campo eléctrico en el centro de la esfera. Hint: Calcule primero, por
definición, el potencial eléctrico en el eje OZ . No intente plantear las integrales del campo por definición.

1
Resolución Auxiliar No2 FI2A2
Prof. Aux.: Felipe L. Benavides
Fecha: Lunes 11 de Agosto

Problema 1

a) Para ésta parte, se escribirán los campos eléctricos originados por cada carga, y con el ppio. de superposición,
se obtendrá una expresión para el campo total, en tres dimensiones. Para cada caso utilizamos Ley de Gauss. Tal
y como antes, dada la geometrı́a de las cargas se asume el campo radial según coordenadas esféricas, y la superficie
encerrada corresponde al manto de la esfera de radio r. Ası́:
− −
→ → QV
Z
E · dS =
S=dV ε0
Z 2π Z π Z 2π Z π
q q
⇒ E(r)r̂ · r̂r2 sen(θ)dθdφ = − ∧ E(r)r̂ · r̂r2 sen(θ)dθdφ =
0 0 ε0 0 0 2ε0
(→

E (r) = − 4πεq0 r2 r̂ (1)
⇒ →
− 0
E (r ) = 8πεq0 r02 r̂0 (2)
Donde (1) corresponde al campo generado por la partı́cula con carga -q, y (2) a aquella con carga + 2q . Al pasar
el campo a coordenadas cartesianas, y usando el mismo sistema de referencia, centrado en -q, para ambos casos, la
superposición entrega, (trabajando sólo la magnitud del campo):
 

− q 1 1
| E (xî + y ĵ + z k̂) |= −
4πε0 2 ((x − a)2 + y 2 + z 2 ) x2 + y 2 + z 2


Al evaluar en el eje x, imponemos que y = z = 0. Agregando la condición | E |= 0,
1 1 √
2
= 2 ⇒ x2 = 2(x − a)2 ⇒ x2 − 4ax + 2a2 = 0 ⇒ x = a(2 ± 2) [m]
2(x − a) x
Lo que resuelve el problema.

b) Queremos probar que existe una superficie equipotencial nula con forma esférica, para lo que planteamos inicial-
mente el potencial genérico. Ası́,
q X qk
V (→
−r)=
4πε0 k→−
r −→ −rkk
k
Entonces,
" #
q 1 1
V (→

r)= 1 − 1
4πε0 2 ((x − a)2 + y 2 + z 2 ) 2 (x2 + y 2 + z 2 ) 2
Es importante notar en ésta expresión que se asume que la referencia es tal que el potencial se anula en el infinito.
Esto es no sólo razonable particularmente, sino que se extiende a cualquier distribución finita de cargas. Eso sı́,
pudiera ser que el potencial fuera nulo en otra(s) posición(es) del espacio, pero aún si ésto sucede, conviene usar lo
antes nombrado. De no hacerlo ası́, la expresión inicial genérica no es válida, pues falta un término constante que
dependerı́a de la referencia usada. Por lo anterior, tiene sentido usar ésta expresión para buscar una superficie a
potencial cero ’con forma’, aún sabiendo que en el infinito existe una, también nula. Imponiendo V = 0,
 12  21 x2 + y 2 + z 2
2 (x − a)2 + y 2 + z 2 = x2 + y 2 + z 2 = ± (x − a)2 + y 2 + z 2


4

1
Al resolver, las soluciones quedan:
( 2
4a 2
(x − 3 ) + y 2 + z 2 = 4a9 (+)
2
4a 2
(x − 5 ) + y 2 + z 2 = − 4a25 (−)
Con ésto, las coordenadas del centro de la esfera obtenida en (+), (pues (-) no tiene significado fı́sico), son:
( 4a
3 0, 0).
,
Problema 2

− −
a) Por la simetrı́a de la situación, se asume que E (→r ) = E(z)(−k̂). Adicionalmente, se supone que la densidad de
carga libre de la atmósfera es constante, (aún cuando no lo sea en ésta situación, pues para efectos de lo buscado es
equivalente), y que ésta se comporta como el vacı́o en términos eléctricos. Planteando la ley de gauss en su forma
diferencial, en coordenadas cartesianas, colocando el origen sobre la superficie terrestre,

− ρ(→
−r) ∂E ρ
∇· E = ⇒− (z) =
ε0 ∂z ε0
El signo menos aparece pues las coordenadas cartesianas tienen sus vectores unitarios definidos positivamente,
(i.e., con orientación hacia afuera del origen), luego el - que aparece en el campo inicial debe trasladarse a su
magnitud. Esto es además coherente, pues los datos confirman que la magnitud del campo disminuye con la altura.
Ası́,
Z z
ρ ρ
E(z) = − dz = C − z
ε
z0 0 ε 0
Con C constante. Evaluando los datos,
( V 
E(0) = 200 m ρ ρ 180ε0
 V  ⇒ E(z) = 200 − z ⇒ 20 = 200 − 1400 ⇒ ρ =
E(1400) = 20 m ε0 ε0 1400
Tomando ε0 ' 8.854 · 10−12 m F
, ρ ' 1.14 · 10−12 mC3
   

b) Para ésta parte, primero se obtiene la carga que debe tener la esfera conductora para alcanzar el campo crı́tico
justo en la vecindad del manto. Con ello, se calculará el potencial. Ası́, (en coordenadas esféricas),
Z 2π Z π

− → − Q →
− − Q
E ( r ) = E(r)r̂ ⇒ E(r)r̂ · r̂r2 sen(θ)dθdφ = ⇒ E (→
r)= 2

0 0 ε 0 4πε 0r

Evaluando, se obtiene la siguiente expresión para despejar Q:


Q
⇒ 3 · 106 =
4πε0 0.12
El potencial crı́tico máximo queda:
0.1
3 · 106 4πε0 0.01
Z
− ˆ
r̂ · (−r)dr = −300000 [V ]
∞ 4πε0 r2
De forma análoga, se resuelve la situación con una esfera conductora de 1 coulomb. Se tendrá, (con el mismo
valor antes usado de ε0 ):
1
3 · 106 = ⇒ r ' 54.73 [m]
4πε0 r2

2
Problema 3

Primero es importante notar que es posible calcular el campo eléctrico en el origen por definición y dicho cálculo
es mucho más corto que el presentado a continuación. Queda propuesto, puesto que la solución por éste camino es
algo más ’compleja’ de obtener, dado que requiere argumentación geométrica más directa. Para obtener el campo
eléctrico, se obtendrá primero el potencial eléctrico, y luego se calculará el gradiente con lo que se tendrá lo pedido.
Por definición, (con referencia de potencial nulo en el infinito),
Z

− 1 1
V(r)= dq(→−r 0)
4πε0 k r −→

− −r0k
con la integral definida en éste caso en la superficie del casquete, cargada. → −r representa la(s) posición(es)

− 0
deseada(s), y r la(s) ubicación(es) de la(s) carga(s). Por conveniencia se centra el casquete en el origen. Como
se busca eventualmente tomar gradiente, no tiene sentido calcular el potencial en un punto. Se calculará, en vez,
el potencial en todo el eje k̂, (en z), y luego se impodrá z=0. (La idea de calcular potencial en cierto eje para
luego tomar gradiente, imponer cierta(s) condición(es) y finalmente obtener el campo eléctrico deseado es utilizada
a menudo). Para escribir el término en el denominador de la fracción dentro de la integral, se observa que:

k→

r −→

r 0 k=k z k̂ − rr̂ k=k z k̂ − rsenθcosϕî − rsenθsenϕĵ − rcosθk̂ k

p
= z 2 − 2zrcosθ + r2
El casquete esférico queda parametrizado por:

r = R

π
 2 ≤θ ≤π
0 ≤ ϕ ≤ 2π

De ésta forma, la expresión queda: (con dq = σdS)


2π π
σR2 senθdθdϕ 2πσR2 π
Z Z Z
1 senθdθ
V (z) = √ = √ =
4πε0 0 π
2
z 2 − 2zRcosθ + R2 4πε0 π
2
z 2 − 2zRcosθ + R2

σR2

2 p 2 2
σR h p i
= − z − 2zRcosθ + R = z + R − z 2 + R2
2ε0 −2zR π 2ε0 z
2

Usando ahora,


E = −∇V
se confirma con la ecuación que el campo eléctrico, por simetrı́a, en el eje z depende sólo de z, (y dada la
disposición de cargas apunta según k̂). Calculando, tenemos:

1 − √RR
!

− σR2 2 +z 2
E (z) = k̂
2ε0 z2


Se desea el campo E (z = 0), pero la expresión anterior no está bien definida allı́. Sin embargo, como al tender z
a cero, tanto numerador como denominador caen a cero, es posible utilizar la regla de l’Hopital. Ası́, (y esto finaliza
el problema):
 
d √ R
σR2 dz 1 − R2 +z 2
 

− →
− σ V
E (z = 0) = lim E (z) = k̂ lim d
= k̂
z→0 2ε0 z→0 2
dz (z )
4ε0 m

3
Enunciado Auxiliar No3 FI2A2
Prof. Aux.: Felipe L. Benavides.
Fecha: Lunes 18 de Agosto de 2008

Problema 1

El espacio entre dos superficies esféricas concéntricas conductoras, (con cargas Q y −Q y radios a y b respectivamente,
a < b), está lleno con dos materiales dieléctricos caracterizados por ε1 y ε2 respectivamente. Estos dos materiales
están separados por un plano ecuatorial. Suponga que los campos son proporcionales a r̂.

a) Obtenga la densidad de carga libre en cada una de las cuatro superficies semiesféricas.

b) Encuentre los valores de desplazamiento eléctrico que caracterizan la esfera.

c) Obtenga la diferencia de potencial entre los cascarones esféricos. ¿Aparecen cargas de polarización? ¿Dónde?
¿Cómo las calcuları́a?
Problema 2
Se desea medir la altura h de un lı́quido, en un recipiente extenso de forma rectangular hecho de vidrio. Para ello, se
colocan tres placas conductoras paralelas rectangulares planas. Entre las placas superior e inferior se establece una
diferencia de potencial V0 mediante una baterı́a. Si la permitividad del lı́quido es ε y la segunda placa conductora se
encuentra descargada, determine el potencial de ésta en función de h. Suponga que el ambiente tiene permitividad
ε0 . (Ver figura 1)

Figura No 1

Problema 3

Asuma la energı́a potencial de interacción entre dos dipolos, (el sistema que constituyen), ubicados en →

r1 y →

r2 ,
conocida, e igual a

→
−p1·→− (→

p1·→

r 21 ) (→

p2·→
− 
1 p2 r 21 )
U= −3
4πε0 k →

r 21 k3 k→
−r 21 k5

1
Con →

r 21 ≡ →

r2−→

r 1 . Utilice éste resultado para resolver la siguiente situación particular:

Dos moléculas de agua, (de momento dipolar p cada una), se encuentran separadas una distancia D. Considere
las siguientes situaciones ((i) y (ii)). En (i), los dos momentos dipolares apuntan en dirección perpendicular a la
lı́nea que une sus centros. En (ii), los momentos dipolares son colineales con dicha recta. En cada caso los dipolos
apuntan en la misma dirección. Dado lo anterior, obtenga:

a) Diferencia de energı́a entre las posiciones i) y ii).

b) Si se les libera desde el reposo, ¿En qué dirección se moverán las moléculas en cada caso? ¿Puede estimarse la
aceleración, velocidad y tiempo de interacción? c) Obtenga la expresión de la energı́a de un sistema compuesto
por dos dipolos. Puede ser útil partir de la expresión del potencial de un dipolo eléctrico. (Propuesto).

2
Resolución Auxiliar No 3
Prof. Aux.: Felipe L. Benavides
Fecha: Lunes 18 de Agosto de 2008

Problema 1

El sistema queda representado por la siguiente figura,

Figura No 1

a) Como las superficies esféricas son conductores, las cargas presentes por enunciado +Q y −Q son, necesaria-
mente, libres. Ası́,
σa 4πa2 = Q y σb 4πb2 = −Q

De allı́, pueden deducirse los valores de las densidades de carga libre presentes, donde se asumen distribuciones
de carga uniformes pues se trata de materiales conductores (lo que permite escribir que las cargas por unidad de
área, que multiplicadas por el área, serán la carga total). La mitad de cada cascarón, tendrá,
(
Q 2 Q
4πa2 2πa = 2 Sup. r = a
Q 2 Q
− 4πb2 2πb = − 2 Sup. r = b

b) Para calcular el vector Desplazamiento Eléctrico, utilizamos ley de Gauss en su forma general para materiales
polares. No podemos utilizar la ley de Gauss en su forma básica para el campo eléctrico pues ésta asume que
se encierran cargas en una superficie no polar, el vacı́o. De argumentos geométricos, y usando que los campos
apuntan según r̂, se obtiene que dentro de la esfera, dentro del primer cascarón conductor esférico, tanto campo
como desplazamiento eléctrico son nulos.

Como se trata de caracterizar la esfera, basta escribir el valor del Desplazamiento entre los cascarones conductores
en cuestión. Al encerrar con una superficie esférica el cascarón conductor de radio a, con una esfera de radio r,
(a < r < b), usamos, en coordenadas esféricas, que


D(r) = D(r)r̂

Ası́,

1
− −
→ →
I
D · dS = QL ⇒
S=dV
Z 2π Z π
D(r)r̂ · r̂r2 senθdθdφ = QL
0 0

Aquı́ debemos detenernos un poco. Como se trata de dos materiales diferentes, (con permitividades dieléctricas
diferentes), en principio no es posible asumir que se trata sólo de un vector desplazamiento eléctrico. Perfectamente
podrı́an haber dos, diferentes, en cada material. La única forma de despejar ésta interrogante es usar condiciones
de borde. Éstas, (que se cumplen en cualquier situación), son;
(
E1t = E2t (i)
D1n − D2n = σl (ii)

Figura No 2

− →

Suponiendo dos materiales según la figura 1, y campos D y E , (i) significa que la componente de campo
eléctrico tangencial a la superficie es equivalente entre ambos materiales, y (ii), que las componentes normales del
desplazamiento entre ambos materiales difieren según la carga libre que haya en la interfaz, según se muestra en la

− →
− →

figura 2. Usando (i), D = ε E (= εr ε0 E ), y evaluando la expresión en la región ecuatorial del sistema,
D1 D2
=
ε1 ε2
Volviendo a la ecuación de Gauss, tendremos,
ε2 Q
D1 (r) + D1 (r) =
ε1 2πr2
Ası́,  

− Q ε1
D 1 (r) = r̂
2πr2 ε1 + ε2
 

− Q ε2
D 2 (r) = r̂
2πr2 ε1 + ε2

c) Para la diferencia de potencial, planteamos la resta de potenciales entre a y b. (Intuitivamente, el potencial


es mayor en la carga positiva, i.e., en a, y menor en b).
a b a b

− → →
− → →
− → →
− →
Z Z Z Z
V (a) − V (b) = − E · d−
r −− E · d−
r =− E · d−
r = E · d−
r
∞ ∞ b a

De las ecuaciones anteriores,



− Q 1
E (r) = r̂
2πr2 (ε1 + ε2 )

2
Finalmente,    
Q 1 1 Q b−a
V (a) − V (b) = − =
2π(ε1 + ε2 ) a b 2π(ε1 + ε2 ) ab
Efectivamente, por la presencia de cargas libres, hay cargas de polarización en el sistema. Dichas cargas se alojan
en la vecindad de los casquetes esféricos y tienen signo opuesto a ellos, mayoritariamente. Pueden ser calculadas
obteniendo el vector polarización y con él, las cargas superficiales y volumétricas de polarización.

Problema 2

Lo primero en éste problema es comprender porqué es posible obtener una expresión para el potencial en función de
la altura de cierto lı́quido, y de allı́ calcular dicho potencial. La idea de la situación es que el potencial en la placa
conductora a altura h depende de los valores del campo eléctrico que están definidos bajo ella. Como el potencial
es cero en la placa inferior, (pues está conectado a tierra), la medición de los campos comienza desde allı́, y se ve
afectada por los materiales que atraviesa.

La baterı́a inicialmente coloca una diferencia de potencial V0 entre las placas superior e inferior. Dicho potencial
está siendo generado por un campo eléctrico que apunta desde las cargas positivas, a las negativas, i.e., según −k̂.
Es intuitivo pensar que si hay un material con permitividad dieléctrica en el camino del campo eléctrico, dicho
campo se verá disminuı́do.

Ésto pues el campo atraerá las cargas negativas hacia arriba, y las positivas, hacia abajo, polarizándolo, y por
lo tanto formando un campo eléctrico por cargas de polarización opuesto al inicial, dentro del material. De ésta
forma, la altura del frasco dirá acerca de la cantidad de lı́quido que “obstruirá” el paso del campo, y por lo tanto,
el potencial en la segunda placa conductora descargada variará según h.

Necesitamos entonces conocer el campo, e integrarlo después. Para ello, planteamos ley de Gauss para materiales
polares, con la carga -Q entre medio, y mediante condiciones de borde despejamos el valor de campo afuera. Lo
primero es notar que el campo eléctrico arriba de la primera placa conductora es cero, y bajo la tercera, también.
Ésto pues el potencial no se ve alterado ni más arriba ni más abajo de las placas, luego el campo debe ser nulo.
Usando un cilindro de radio r con altura y disposición cualquiera tq. se encuentre dentro del material su tapa
superior, y bajo la placa conductora inferior la tapa opuesta, se tendrá, (asumiendo campos y desplazamiento a lo
más dependientes de z, y apuntando según -k̂),


D(r) = −D(z)k̂

− −
→ →
I
D · dS = QL
S=dV

Sólo las tapas superior e inferior constituyen aportes, pues el manto tiene normal perpendicular al desplazamiento
eléctrico. La tapa de arriba dentro del material entrega un valor, pero la que se encuentra bajo la placa conductora
inferior no, pues notamos ya que el campo eléctrico era nulo. En coordenadas cilı́ndricas,
Z 2π Z r Z 2π Z r
D(z)ρdρdθ = −σρdρdθ
0 0 0 0

−Dπr2 = −σπr2 ⇒ D = σ
Ası́, con la magnitud del desplazamiento eléctrico obtenida,


D(r) = −σ k̂ ⇒


− σ
E (r) = − k̂
ε
Dentro del material. Afuera, podemos usar ley de gauss denuevo, pero para la placa superior ésta vez. La otra
opción, es simplemente usar condiciones de borde. Como la interfaz, definida como la capa que separa el lı́quido del

3
aire, no está cargada, los vectores desplazamiento eléctrico son iguales, y con ello,(D1n − D2n = σl ⇔ D1 = D2 ),
afuera del material,

− σ
E = − k̂
ε0
Con los campos listos, basta despejar la expresión para el potencial en h. El único problema es que σ es
inicialmente desconocido, (no es dato del problema), aunque es calculable. De plantear que el potencial entre las
placas superior e inferior es V0 , puede despejarse dicha constante. Ası́, (distinguiendo con números los campos para
destacar que no son iguales),
h h+d d+a
− →
→ − − →
→ − − →
→ − σh σ(d − h) σa
Z Z Z
V0 = − E 1 · dr + − E 2 · dr + − E 2 · dr = + +
0 h d ε ε0 ε0
V0 εε0
σ= (∗)
hε0 + (d − h + a)ε
Ası́,
h d
− →
→ − − →
→ −
Z Z
σ σ
V (h) = − E 1 · dr + − E 2 · dr = h + (d − h) (∗∗)
0 h ε ε0
Reemplazando (*) en (**),
 
hε0 + (d − h)ε
V (h) = V0
hε0 + ε(d − h + a)

Problema 3

Planteando las situaciones descritas y la energı́a asociada según la fórmula,

i.


r 21 = →

r2−→

r 1 = Dx̂, →

p1 =→

p 2 = pŷ

1 p2
Ui =
4πε0 D3

ii.


r 21 = →

r2−→

r 1 = Dx̂, →

p1 =→

p 2 = px̂

p2 p2 D 2 p2
 
1
Uii = 3
−3 5 =−
4πε0 D D 2πε0 D3
Ası́,

3 p2
Ui − Uii =
4πε0 D3
b) Si se liberan desde el reposo, la energı́a tenderá a disminuı́r, luego las distancias entre sus centros serán
mayores o menores según las ecuaciones muestren. En i, la energı́a disminuye si D aumenta, por lo que los dipolos
se repelerán. En ii, la energı́a disminuye si D disminuye, por lo tanto, los dipolos se atraerán. En efecto, puede
estimarse aceleración, velocidad y tiempo de interacción, por ejemplo, modelando como un sistema newtoniano, y
utilizando que
∂Ui ∂Uii
k Fi k=k k, k Fii k=k k
∂D ∂D

4
Enunciado Auxiliar No4 FI2A2
Prof. Aux.: Felipe L. Benavides
Fecha: Lunes 25 de Agosto de 2008

Problema 1

En el centro de una cavidad esférica de radio a practicada en un bloque de material dieléctrico de permitividad
relativa k, se coloca una carga puntual q. Calcule el potencial eléctrico en todos los puntos del espacio. Demuestre
además que la suma de las cargas inducidas y la carga original es kq , independiente de a.

Problema 2

Se tienen dos planos conductores paralelos infinitos de ecuaciones x = d, y x = −d, respectivamente. El plano
x = −d está conectado directamente a tierra y el otro, a través de una baterı́a cuya diferencia de potencial entre
sus bornes es V0 . La región comprendida entre los planos está rellena con un material dieléctrico, cuya constante
dieléctrica es:
4
ε = ε(x) =
x 2

d +1

Calcule:

a) Campo eléctrico en todo el espacio, en función de la densidad de carga σ de los planos.

b) Función potencial en todo el espacio.

c) Densidad de carga superficial σ en cada uno de los planos en función de V0 .



b) Vector polarización, P .

c) Densidad de carga de polarización, ρp , y densidad de carga superficial de polarización, σp , en las dos superficies
del dieléctrico.

Problema 3, Propuesto

Una esfera de radio R de material con permitividad dieléctrica ε0 está cargada con densidad de carga uniforme, de
tal modo que su carga total es q.

a) Calcule la energı́a del sistema.

b) Suponga que la esfera se trata de un electrón. La energı́a de éste, obtenida según consideraciones relativistas,
es del orden de 10−13 Joule. Obtenga el orden de magnitud del radio del electrón.

1
Resolución Auxiliar No 4 FI2A2
Prof. Auxiliar: F. L. Benavides
Fecha: Lunes 25 de Agosto de 2008

Problema 1

Para calcular el potencial eléctrico en todo el espacio es necesario obtener el campo eléctrico para todas las regiones
del sistema. Poniendo el origen en la posición de la carga q, por casos, se obtiene: (Ojo que el dato es k permitividad
relativa. Ella es tal que ε0 k = ε permitividad dieléctrica del material).

r ≤ a;

Z 2π Z π
q
E(r)r2 sen(θ)dθdφ = ⇒
0 0 ε0


− q
E (r) = r̂
4πε0 r2

r > a;


− q →
− q
D(r) = r̂, E (r) = r̂
4πr2 4πε0 kr2

El potencial eléctrico queda: (con la referencia de potencial nulo en el infinito)

r
− →
→ −
Z
− E · dr ⇒

Si r > a,

Z r  r
q q 1 q
V (r) = − 2
· dr = =
∞ 4πr ε0 k 4πε0 k r ∞ 4πε0 kr

Si r ≤ a,

r a r a r
− →
→ − − →
→ − − →
→ −
Z Z Z Z Z
q q
V (r) = − E · dr = − E · dr − E · dr = − · (dr) − · (dr) ⇒
∞ ∞ a ∞ 4πε0 r2 a 4πkε0 r2

1
 
q 1 1 1
V (r) = − + +
4πε0 ak a kr

Eso por un lado. Para demostrar el resultado que se pide en relación a las cargas inducidas, primero es necesario

− →

obtener el vector de polarización. De la definición, P = (ε − ε0 ) E , se llega a:

(
q(k−1)

− 4πkr 2 r̂ r>a
P (r) =
0 r≤a

De aquı́ que:


− 1 ∂
ρp = −∇ · P = 2 (P · r2 ) = 0
r ∂r

h→
− i q(k − 1)
σp = P · n̂ =−
r=a 4πka2

De ésta forma, al sumar la carga total de polarización superficial junto con la puntual en el origen, vemos que:

q(k − 1) q
− +q =
k k

Es decir, la carga total no depende del radio, tal y como se deseaba probar.

Problema 2

Primero notamos que el campo eléctrico para |x| > d es nulo. Esto pues, dada la descripción del sistema, la diferencia
de potencial entre cualquier par de puntos al lado izquierdo o derecho del eje es cero, i.e., no puede haber campo.
En general, cuando hay planos infinitos se asume éste resultado, que no depende del tipo de dieléctrico que haya
entre medio. Puede probarse más detalladamente calculando los efectos producidos por cada conductor cargado, y
luego sumando. La idea teórica detrás de ésto es que una lı́nea de campo nunca es abierta, todas parten y terminan
en cargas, (positivas y negativas respectivamente). Si hubiese lı́neas de campo hacia la derecha o izquierda, tendrı́an
que devolverse en el infinito, pero dado que son rectas, (pues los conductores son infinitos), ésto no puede ocurrir.

a) Por ley de Gauss, utilizando una superficie cerrada cilı́ndrica entre el medio de la derecha, la placa conductora
con carga +σ, (x = d), y el medio en cuestión, sólo influye la cara del cilindro que queda encerrada en x < d. (El
manto tiene normal perpendicular y el campo para x > d es nulo). Entonces,

− −
→ → →

Z Z
D · dS = σdS ⇒ −D4S = σ4S ⇒ D = −σ î ⇒

2
h i
x 2
(
−σ


− 4ε0 d + 1 î |x| < d
E (x) =
0 |x| > d

b) Para la función potencial, calcularemos primero dentro del dieléctrico, y luego concluiremos para todo el espacio.
Ası́,

|x| < d,

x   3
1 x + d3
   
−σ  x 2
Z
σ
V (x) = − + 1 dx = + (x + d)
−d 4ε0 d 4ε0 d2 3

 h  3 3 i
σ 1 x +d

 4ε0 d2
 3 + (x + d) |x| < d
∴ V (x) = V0 x>d


0 x<d

c) Las densidades de carga superficial son iguales en magnitud y opuestas en signo. Basta que igualemos el potencial
en x = d con las dos expresiones que se tienen, y se despeja la densidad superficial. Ojo, que el campo no está
definido para x = d, no ası́ el potencial, que es la integral del primero. Como la discontinuidad es finita, no hay
complicaciones.

  3
1 d + d3
 
σ 3V0 ε0
2
+ (d + d) = V0 ⇒ σ =
4ε0 d 3 2d

(Para −σ basta cambiar el signo).

d) El vector polarización queda de la siguiente forma:


− →

P = (ε − ε0 ) E ⇒

 
x 2
(
−σ σ


− ε0 + 4 d +1 |x| < d
P (x) =
0 |x| > d

e) Las densidades de carga de polarización quedan de la siguiente forma:


− σx
ρp = −∇ · P = − 2
2d

3
 

− 1 1
σp = P · n̂ = σ −
2 ε0

Donde el σp calculado corresponde al efecto a la derecha. (A la izquierda es simplemente el mismo, con signo
contrario, pues las normales son opuestas.

Problema 3

Éste problema queda propuesto puesto que es simplemente ilustrativo respecto de las aplicaciones de la teorı́a
estudiada a la realidad. Para el cálculo de la energı́a del electrón, se propone simplemente realizar:


− → −
Z
1
E= E · D dV
2

Donde los vectores campo y desplazamiento eléctrico son muy sencillos de calcular, para la carga q dada. Luego,
la idea es comparar el valor según consideraciones relativistas, teóricas, de 10−13 [J], para un electrón, y con ello
estimar el orden de magnitud de su radio.

4
Enunciado Auxiliar No5 FI2A2
Prof. Auxiliar: Felipe L. Benavides

Fecha: Lunes 1 de Septiembre de 2008

Problema 1

Considere un sistema infinito de simetrı́a cilı́ndrica compuesto, al centro, por un alambre rectilı́neo con densidad de
carga λ0 uniforme, rodeado de un cilindro de radio a, de material dieléctrico con constante dieléctrica εa , a su vez
rodeado de un cilindro conductor de radio exterior c el cual, finalmente, está rodeado de un cilindro dieléctrico de
radio exterior b y de constante dieléctrico εb . El cilindro conductor está cargado; su carga por unidad de longitud
es λ1 . (Cada “anillo”). Determine el campo eléctrico y de desplazamiento en todas partes.

Problema 2

Considere 2 casquetes esféricos concéntricos conductores de radios a y b, con a < b. El conductor de radio a está a
un potencial V0 , y el conductor de radio b está conectado a tierra. (Potencial cero). El espacio entre los conductores
se rellena con un dieléctrico de permitividad ε, según la figura 1. Calcule el campo y desplazamiento eléctrico,
vector polarización, densidades de carga de polarización superficial y volumétrica, entre los casquetes. (La carga Q
es desconocida).

Figura No 1.

Problema 3
-Condensadores-

Situación I. La permitividad de cierto material varı́a linealmente de una placa a otra en un condensador de placas
paralelas. Si ε1 y ε2 son los valores de la permitividad de cada una de las placas, y ε1 < ε2 , determine la capacidad
Q
por unidad de área, siendo d la separación de las placas. (Recuerde: capacidad C = 4V ).

Situación II. Considere un sistema de electrodos rectangulares de área unitaria, al cual se le aplica una diferencia
de potencial V0 . Entre las placas se coloca una lámina conductora ideal. Determina las densidades de carga libre
que aparecen en las caras de la lámina central.

1
Solución Auxiliar No 5 FI2A2
Prof. Aux.: Felipe L. Benavides
Fecha: Lunes 01 de Septiembre de 2008

Problema 1

Éste problema es ciertamente sencillo. Sólo es importante tener cuidado con los efectos en los
bordes. Dada la geometrı́a del sistema, podemos asumir que el campo tiene simetrı́a cilı́ndrica, con
dependencia sólo en ρ, y que apunta según ρ̂. Con lo anterior, en cada caso basta plantear Gauss
con campo o desplazamiento eléctrico, (según corresponda), y concluir. Se tendrá que:

ρ < a;

Z z0 Z 2π Z z0
D(ρ)ρdθdz = λ0 dz ⇒
0 0 0


− λ0 →
− λ0
D(ρ) = ρ̂, E (ρ) = ρ̂
2πρ 2πρεa

a < ρ < c;


− →

D(ρ) = E (ρ) = 0

c < ρ < b;

Z z0 Z 2π Z z0 Z z0
2 2
D(ρ)ρdθdz = λ0 dz + π(c − a ) λ1 dz ⇒
0 0 0 0


− λ0 + λ1 π(c2 − a2 ) → − λ0 + λ1 π(c2 − a2 )
D(ρ) = ρ̂, E (ρ) = ρ̂
2πρ 2πρεb

ρ > b;

z0 2π
R z0 R z0
λ0 dz + π(c2 − a2 ) λ1 dz
Z Z
0 0
E(ρ)ρdθdz = ⇒
0 0 ε0

1

− λ0 + λ1 π(c2 − a2 ) → − λ0 + λ1 π(c2 − a2 )
D(ρ) = ρ̂, E (ρ) = ρ̂
2πρ 2πρε0

ρ = a, c;


− →

D(ρ)@, E (ρ)@

Es importante notar que se tuvo cuidado con los bordes de cada volúmen, en el caso del volúmen
conductor. Como el campo es cero justo dentro de él, dadas sus propiedades, (reacomodo de cargas
frente a un campo eléctrico cualquiera), y distinto de cero justo después, no es posible definirlo en
la interfaz. Observamos también que en ρ = b el vector desplazamiento es contı́nuo, pero el campo
no. Esto se debe a que en la superficie de separación entre el vacı́o y el dielétrico, aparece una
densidad de carga de polarización, no carga libre. (Lo que es coherente con la condición de borde
D1n − D2n = σs )

Problema 2

Para resolver éste problema, se utilizará gauss nuevamente, asumiendo que dada la geometrı́a del
sistema, el campo depende sólo del radio, en coordenadas esféricas, y apunta según r̂. Ésta suposición
es algo fuerte, pero puede fundamentarse pues el dieléctrico es coherente con la esfera en términos
de que la superficie tangencial, entre los casquetes conductores, se orienta según r̂. Ası́, llamando
I al volúmen dentro del dieléctrico, y II al restante entre los casquetes, basta despejar el valor del
campo y usar las fórmulas conocidas para todo lo demás pedido. Queda: (entre los casquetes)

a < r < b;

Z 2π Z π
D(r)r2 sen(θ)dθdφ = Q ⇒
0 0

Z 2π Z α Z 2π Z π
D1 (r) r2 sen(θ)dθdφ + D2 (r) r2 sen(θ)dθdφ = Q ⇒
0 0 0 α

D1 (r)2πr2 (1 − cos(α)) + D2 (r)2πr2 (1 + cos(α)) = Q

En éste punto debemos usar alguna ecuación que relacione ambos desplazamientos. De las condi-
ciones de borde, se tiene que:

(
D1n − D2n = σs (1)
E1t = E2t (2)

2

− →

Ambas condiciones son, en general, linealmente independientes. Sin embargo, dado que D = ε E ,
con ε conocido en ambos lados, necesitamos una sola de ambas. Utilizaremos la expresión (2) por
simplicidad, pero la primera, más la definición del vector desplazamiento, también resolverı́an el
problema. (Sólo que habrı́a que usar consideraciones entre el volúmen dentro de los casquetes,
y aquel afuera, o dentro del casquete conductor pequeño, radio a. Usando lo recién nombrado,
tenemos:

D1 D2
= ⇒
ε ε0


− Q →
− Q
D 1 (r) = ε0
 r̂, D 2 (r) =   r̂ y
2πr2 1 − cos(α) + ε (1 + cos(α)) 2πr2 1 + cos(α) + ε
− cos(α))
ε0 (1


− Q
E (r) = r̂
2πr2 (ε0 (1 + cos(α)) + ε(1 − cos(α)))

Notamos a ésta altura que la carga Q, según el enunciado, es desconocida. Esto no es una compli-
cación, pues con la expresión recién obtenida del campo, basta calcular el potencial e igualar con
el dato V0 del problema. Queda: (usando que g0 = ε0 (1 + cos(α)) + ε(1 − cos(α)))



r
− →
→ −
Z
− E · dr = V (→

r)⇒


r0

Z a
Q
− dr = V0 ⇒
b 2πr2 g0

2πg0 V0 ab
Q=
b−a

Para la polarización, separamos por casos:

∀θ ∈ [0, α], φ ∈ [0, 2π] y r ∈ (a, b);


− (ε − ε0 )Q
P = r̂
2πr2 g0

∀θ ∈ [α, π], φ ∈ [0, 2π] y r ∈ (a, b);


− →

P = (ε0 − ε0 ) E = 0

3


Las densidades de carga de polarización quedan: (Para el primer caso con P no nulo)


− 1 ∂
ρp = −∇ · P = − 2 (P · r2 ) = 0
r ∂r

( (ε−ε )
− 2πa2 g00 r=a
σp = (ε−ε0 )
2πb2 g0 r=b

Problema 3

Situación I.

Primero planteamos la expresión lineal para la permitividad del dieléctrico. Se tiene:


ε2 − ε1
ε(z) = z + ε1
d
Buscamos
Q
C= ,
4V
pues con ésta expresión, la capacidad por unidad de área es simplemente lo mismo sobre el
área total. (Asumiremos conocido Q magnitud de la carga de las placas conductoras del capacitor,
y calcularemos respecto del éste valor algo del estilo 4V (Q), para después simplemente dividir.
Usando Gauss en el electrodo superior con una superficie cerrada cilı́ndrica, imponiendo que el
campo arriba de la placa conductora es nulo y que apunta hacia abajo, queda:

− → −
Z
D · dS = Q ⇒


− Q →
− −Q
−D4S = Q ⇒ D = − k̂ ⇒ E (z) = ε2 −ε1
 k̂
4S 4S d z + ε1
Integrando el campo para obtener el potencial,
d
−Q
Z
− ε2 −ε1
 dz = V0 ⇒
0 d z + ε1 4S
Q 4S(ε2 − ε1 )
=C=  
4V dln εε21

Por lo tanto la capacidad por unidad de área pedida es:

(ε2 − ε1 )
C=  
dln εε21

Situación II.

Queda propuesto. La versión inicial es simplemente demostrativa, la idea es que se pruebe con
distintas configuraciones, para distancias diferentes entre la lámina conductora y las placas superior
e inferior, y para diferentes permitividades dieléctricas.

4
Enunciado Auxiliar No6 FI2A2
Prof. Aux.: Felipe L. Benavides
Fecha: Lunes 22 de Septiembre de 2008

Problema 1

Ley de Ohm y Resistencias

Para la figura 1, asumiendo que se trata de cascarones esféricos concéntricos, y suponiendo conocidos los datos
allı́ presentados, (salvo V0 que es innecesario para el resultado final), ¿cuál es el valor de la resistencia R entre los
conductores?

Figura No 1

Problema 2

Ley de Ohm, Resistencias y Leyes de Kirchoff

La llamada “protección diferencial” de los sistemas de suministro eléctrico domiciliarios, consiste en un interruptor
automático que funciona desconectando el circuito cuando la corriente que entra es diferente de la corriente que
sale. Supongamos que se tiene conectada una estufa eléctrica de una potencia 2[kW ], que tiene un defecto tal que
el punto medio de la resistencia calefactora está conectado a la caja metálica de la estufa. Si una persona toca la
caja, y además está conectada a tierra a través de sus pies, va a pasar una corriente por la persona. Se pide calcular
cual es la diferencia de corriente entre los dos cables de alimentación de la estufa, sabiendo que la persona presenta
una resistencia entre una mano y sus pies, de 10[kΩ].

Figura No 2

1
Problema 3

Ecuación de Continuidad

Considere un condensador de placas planas paralelas cuyo espacio interno se llena con dos bloques de materiales
distintos, cuyas permitividades y conductividades valen respectivamente (ε1 , g1 ), y (ε2 , g2 ), como se indica en la
figura 3.

a) Si entre las placas se establece una diferencia de potencial V0 , calcule la densidad de cargas libres superficiales
en la región interfacial en régimen permanente.

b) Suponga ahora que, después de haberse establecido el régimen permanente, se desconecta la baterı́a y cada
placa se conecta a tierra. Determine la evolución temporal de la densidad de carga en la región interfacial.

Figura No 3

Problema 4

Fuerza de Lorentz

El diagrama de la figura 4 representa un dispositivo para medir las masas de iones. Un ión de masa m y carga +q
sale de la fuente F prácticamente en reposo. Luego, el ión es acelerado por una diferencia de potencial V y se le


permite entrar a una región de campo magnético de módulo | B |. (El campo sale de la hoja). En presencia de
éste campo se mueve en un semicı́rculo, incidiendo sobre una placa fotográfica a una distancia x desde la rejilla de
entrada. Obtenga expresiones, en términos de B, q, V y x, para: la masa m del ión, el tiempo que se encuentra
dentro del sistema y la posición en función del tiempo.

Figura No 4

2
Resolución Auxiliar No6 FI2A2
Prof. Aux.: Felipe L. Benavides
Fecha: Lunes 22 de Septiembre de 2008

Problema 1

Éste tipo de problemas es muy común, y su solución por tanto también es bastante estándar. La idea es calcular la
corriente en función del potencial V0 al que está sometido el material, y de allı́ simplemente despejar la fracción VI0 .


Se asume por supuesto I constante, no ası́ J , que depende de la posición para permitir que lo anterior se cumpla,

− →

(se trata de régimen permanente). Con ella entonces, después de despejar J , se obtiene E , y con él, V0 . Dada la
disposición geométrica del sistema, el vector densidad de corriente apunta según r̂. Resolviendo entonces,
Z 2π Z π

− → −
Z
I= J · dS = J(r)r2 sen(θ)dθdφ
0 0


− I
I = 4πJ(r)r2 ⇒ J (r) = r̂
4πr2
Ahora, por ley de Ohm,

− →
− →
− I
gE = J ⇒ E = r̂
4πgi r2
Con i=1,2. Como la densidad de corriente es contı́nua en el espacio, (ppto. probarlo en éste caso, aunque es muy
sencillo), la expresión se mantiene y sólo varı́an los g de la conductividad de los materiales. Calculando entonces la
diferencia de potencial, se obtiene:

V0 = V (a) =

a c b Z c

− → →
− →
Z Z Z
I I
=− E · d−
r = E · d−
r = 2
dr + 2
dr ⇒
c a a 4πg 1 r b 4πg 2r
   
I 1 1 I 1 1
V0 = − + −
4πg1 a b 4πg2 b c
   
V0 1 1 1 1 1 1
∴R= = − + −
I 4πg1 a b 4πg2 b c

Problema 2

Para resolver éste problema se plantea un circuito equivalente donde se divide la resistencia del calefactor en 2
trozos, y se conecta, en paralelo con la de más abajo, una equivalente a la persona. Por notación, usando que I1 es
la corriente que circula por el primer trozo de la resistencia, I2 el que circula por la persona, e I3 , por el segundo
trozo, se pide conocer entonces:
I2 = I1 − I3
Para ello, se plantean las siguientes ecuaciones, (de Ohm y Kirchoff):
I1 R I3 R
(LV K) 220 = +
2 2

1
I3 R
(LV K) = I2 · 10000
2

(LCK) I1 = I2 + I3

Para despejar el sistema falta una ecuación, pues R es desconocido. Aquı́ usaremos que:

V2
= 2000 (∗)
R
Es decir, planteamos la expresión de potencia para el calefactor. Según el enunciado, podemos usar que la
potencia consumida es nominal, y no es aquella referida al momento de la falla. El cálculo utilizado para ésto
es exacto, y la resolución tiene total precisión. Si la potencia referida en el enunciado se tratase del consumo al
momento de producirse la falla, la última expresión es sólo una aproximación. Dado que la corriente que circula
por la persona es pequeña, (pues la resistencia es muy grande), no es una mala aproximación. Debieran sumarse
todos los otros términos, pero sólo consideran corrientes pequeñas elevadas al cuadrado, i.e., valores ciertamente
despreciables frente a la primera componente. Por ello, el procedimiento es equivalente en ambos casos, salvo que en
el primero es exacto y en el segundo es una aproximación. (Aunque razonable. Sumar todos los términos también
resuelve el problema, sólo que el despeje del sistema es muy largo y complejo). Resolviendo, tendremos:
220 · 2
I2 =
R + 4 · 104
De (*), R = 24.2[Ω] ⇒

I2 ∼
= 0.011 = 11[mA]
El valor confirma que la aproximación no es mala, pues efectivamente I2 es pequeña, dado que la resistencia del
calefactor es mucho menor que la de la persona. (En órdenes de magnitud incluso).

Problema 3

a) Para encontrar la densidad de carga libre en la interfaz, podemos usar



− →

( D 2 − D 1 ) · n̂ = σl (1)

En régimen permanente, dado que los materiales exhiben una conductividad no nula, hay corriente. Por con-
tinuidad, (conservación de la carga):

− ∂ −
∇ · J = − ρ(→
r , t)
∂t
La condición de régimen permanente queda:
∂ →
ρ(−
r , t) = 0
∂t
Ası́


J (z) = −J k̂
Tomando positivo z de izquierda a derecha, según el dibujo de la figura en el enunciado. Como adicionalmente,
(por ley de Ohm):

− →

J = gE ⇒


− J →
− J
E 1 = − k̂; E 2 = − k̂ ⇒
g1 g2

− J →
− J
D 1 = − ε1 k̂; D 2 = − ε2 k̂
g1 g2

2
Pero a priori no es conocida ni la corriente, ni su densidad. Por ello, necesitamos encontrar una expresión que
ligue los datos restantes del problema con la información ya construı́da. En efecto, por potencial, tendremos:
d1 +d2

− →
Z
− E · d−
r = V0 ⇒
0

−E1 d1 − E2 d2 = V0 ⇒

J J
d1 + d2 = V0 ⇒
g1 g2
V0 g1 g2
J=
g1 d2 + g2 d1
Reemplazando éste último valor en los vectores desplazamiento antes construı́dos y con ello despejando (1):
V0 g1 ε2 V0 g2 ε1
− + = σl
g1 d2 + g2 d1 g1 d2 + g2 d1
V0
∴ (g2 ε1 − g1 ε2 ) = σl
g1 d2 + g2 d1

b) Ésta nueva situación es de régimen transitorio. Para la interfaz, se tiene:


 
∂ → ∂
ρ(−
r , t) 6= 0 ⇒ J2n − J1n = − σl
∂t ∂t

La idea es despejar de ésta ecuación el término para la derivada parcial de la densidad de carga libre en la
interfaz. Para ésto, se necesitan dos ecuaciones que liguen a los términos J2n y J1n , y que dicha relación, (ojalá),
exhiba términos en función de σl , para luego simplemente resolver la EDO que resulte. Nuevamente, (sólo que con
el cuidado de trabajar con dos densidades no necesariamente iguales),

− J1 → − J2
E 1 = − k̂; E 2 = k̂
g1 g2
Como el potencial es igual en ambos lados, (pues están conectados a tierra),

− →

Z Z
J1 d1 J2 d2
= (− E 1 · d→−r = − E 2 · d→ −
r ) (2)
g1 g2
Es importante destacar aquı́ que el potencial entre la interfaz y las placas conductoras depende del tiempo (pues
está conectado a tierra, luego debe descargarse). Por ello no es posible igualar a cero dichos cálculos. Por supuesto,
los campos dependen del tiempo, pero no de la posición. Si hubiera una dirección preferencial de carga/descarga,
habrı́a un campo eléctrico aplicado distinto de cero en la vecindad de dicha posición, lo que genera una contradicción
con lo recién supuesto de simple conexión a tierra.
Nuevamente,

− →

( D 2 − D 1 ) · n̂ = σl ⇒

J2 ε2 J 1 ε1
+ = σl (3)
g2 g1
De (2) y (3),
 
ε1 ε2 d1
J1 + = σl
g1 g1 d2
Reemplazando finalmente éste último valor en cada uno de las densidades de corriente para la interfaz, (ecuación
de continuidad), y utilizando adicionalmente (2),

3
 
∂ g1 d2 + g2 d1
− σl = σl
∂t ε1 d2 + ε2 d1

∴ σl (t) = σl (0)e−λt
Con
 
g1 d2 + g2 d1
λ= .
ε1 d2 + ε2 d1

Problema 4

La resolución de éste ejemplo plantea inicialmente un sistema de ecuaciones. Se cumplen, por conservación de
energı́a y de la ecuación de fzas. de Newton,
(1 2
2 mv = qV (1)
mv 2
= qvB (2)
( x2 )

Donde v corresponde a la rapidez de la partı́cula, θ̇ x2 . De aquı́, resolviendo,

1 B 2 x2 q
m=
8 V
Tanto el tiempo dentro del sistema como la función posición instantánea son muy sencillas de deducir a partir
de éste punto. La última queda definida despejando θ̇ de las ecuaciones. Se obtiene que:
     

− x x 8V 8V
r (t) = ρ̂ = cos t î + sen t ĵ
2 2 Bx2 Bx2
Imponiendo θ(t∗ ) = π,

πBx2
t∗ =
8V
Durante la resolución se supuso, según el enunciado, que la trayectoria es un arco de circunferencia, sin probarlo.
La demostración es muy simple y queda propuesta.

4
Enunciado Aux. No7 FI2A2
Prof. Aux.: Felipe L. Benavides
Fecha: Lunes 29 de Septiembre de 2008

Problema 1

Aproximación Filiforme de B y Ley de Ampére




Para la figura de más abajo, ¿cuánto debe valer r tal que el campo magnético B en el interior, justo en el centro
del cı́rculo, sea nulo?

Figura No 1

Problema 2

Continuidad en Régimen Permanente

La figura de más abajo representa un conductor semicilı́ndrico, radio interno a, radio externo b, largo h, con


conductividad g. Una densidad de corriente J fluye entre los contactos rectangulares A y B. (Asumiéndolos


conductores perfectos). Suponga que J , en coordenadas cilı́ndricas, es proporcional al vector unitario θ̂, propio de


éstas coordenadas, y cuya magnitud depende de la coordenada ρ, tal que J = j(ρ)θ̂.

a) Determine la dependencia de j(ρ) en el radio ρ. (Sólo la forma)

b) Obtenga la corriente total I que fluye entre A y B. (Exprésela en términos de lo calculado en a))



c) Obtenga la diferencia de potencial V0 que hay entre A y B y resuma con V0 como dato los valores para J ,


E e I. Calcule adicionalmente la resistencia R del sistema.

Figura No 2

1
Problema 3

Aplicaciones Ley de Ampére

Se tiene un cable coaxial ideal de simetrı́a cilı́ndrica que consta de un conductor cilı́ndrico macizo de radio a
rodeado por un conductor cilı́ndrico hueco de radio interior b y radio exterior c = 3a. Por el cilindro central pasa


una densidad de corriente uniforme J 1 = J0 k̂ y por el cilindro hueco exterior circula una densidad de corriente


opuesta, J 2 = − 15 J0 k̂. Determine el campo magnético en todas partes y calcule qué valor debe tener b para que
éste último, en la zona exterior, (ρ > c), sea nulo.

Figura No 3

Problema 4

Superposición de Campos Magnéticos y Ley de Ampére

Se tiene un conductor en la forma de una capa cilı́ndrica recta, infinita, de radio interior a y radio exterior b. Este con-
ductor tiene una densidad de corriente que, expresada en coordenadas cilı́ndricas, es:

− α
J (a ≤ ρ ≤ b) = θ̂ + β k̂
ρ
Con α y β constantes conocidas. Obtenga el campo magnético en todas partes.

Figura No 4

2
Resolución Aux. No 7 FI2A2
Prof. Auxiliar: Felipe L. Benavides
Fecha: Lunes 29 de Septiembre de 2008

Problema 1

Para resolver el ejemplo calcularemos el campo magnético producido por cada uno de los componentes del sistema en
el centro del cı́rculo, y luego sumaremos. (Aplicando superposición). De ésta forma, tendremos que para el alambre cir-
cular, por definición, (usando la aproximación filiforme pues se trata de un cable),
Z →

− → − µ0 I d−r × (→−r −→ −
r 0)
B( r ) =
4π Γ k → −r −→ − 0
r k3

Donde, como siempre, → −


r indica la posición en la que se mide, y →−r 0 recorre la carga. (En éste caso el flujo de
corriente, constante). Parametrizando para el centro del cı́rculo, (colocando allı́ el orı́gen de preferencia para efectuar
el cálculo, y teniendo cuidado con el signo del diferencial dada la forma en que se recorre la curva),

−Rdθθ̂ × (0k̂ − Rρ̂)
Z

− µ0 I
B (0) =
4π 0 R3

µ0 I R2 µ0 I
=− 3
2π k̂ = − k̂
4π R 2R
Para el alambre rectilı́neo, usamos por conveniencia, la ley de Ampere, conociendo la dirección del campo previa-
mente por el uso de la regla de la mano derecha. (Ésta es general y muy útil para encontrar la dirección de los campos
en situaciones sencillas). Se tiene que, nuevamente, parametrizando para el centro del cı́rculo, (ésta vez colocando el
origen en la proyección r de radio del cı́rculo justo sobre el alambre rectilı́neo),
Z
− →
→ −
B · d l = µ0 I ⇒

Z 2π
B(ρ)ρdθ = µ0 I ⇒
0


− µ0 I
B (ρ) = θ̂
2πρ
Vemos que para calcular la suma de los campos no es necesario adaptar los orı́genes, pues, según se tomaron
inicialmente, los vectores unitarios θ̂ y k̂ coinciden. Evaluando en ρ = r, y planteando el campo magnético total, se
obtiene:
 

− µ0 I 1 1
B T (0) = − k̂
2 πr R

R
∴r=
π
Es el valor de r que anula el campo magnético total en el punto deseado.

1
Problema 2

a) De la ecuación de continuidad para la corriente en régimen permanente,




∇· J =0

Como

− →

J = gE

Y por lo anterior,


∇× E =0⇒

1 ∂(ρJ)
=0⇒
ρ ∂ρ


− C
J (ρ) = θ̂
ρ
Con C alguna constante por despejar.
b) Por definición, usando (a), (para alguna sección transversal del sistema),
Z Z h Z b  

− → − C b
I= J · dS = θ̂ · θ̂dρdz = Chln
0 a ρ a

c) La diferencia de potencial V0 se obtiene tb. por definición, como la integral de lı́nea del campo. Ası́,
Z π

− →
− →
− C C πC
g E = J ⇒ E (ρ) = θ̂ ⇒ V0 = ρdθ =
gρ 0 ρg g

Usando V0 como dato y resumiendo, se obtiene:


 

− V0 g → − V0 V0 gh b V0 π
J (ρ) = θ̂, E (ρ) = θ̂, I = ln , =R= b

πρ πρ π a I ghln a

Problema 3

Éste ejemplo es una sencilla aplicación de la Ley de Ampere. Para cada trozo, sabemos a priori que el campo
magnético apunta según θ̂, (regla de la mano derecha), y que no depende más que de ρ. El sistema es infinito en
z y simétrico, (dadas las simetrı́as en θ), por lo que podemos afirmar que lo anterior se cumple en todos los casos.
Resolviendo,
Z
− →
→ −
B · d l = µ0 I
Γ

Donde I es la corriente que cruza la superficie encerrada en la curva Γ, cerrada, que recorre la integral de lı́nea.
Ası́,
ρ≤a
Z 2π Z 2π Z ρ
ρ2
Z

− → −
B(ρ)ρdθ = µ0 J · d S = µ0 J0 ρ0 dρ0 dθ = µ0 J0 2π ⇒
0 0 0 2


− µ0 J0 ρ
B (ρ) = θ̂
2
a≤ρ≤b

2
Z 2π Z

− → −
B(ρ)ρdθ = µ0 J · d S = µ0 J0 πa2 ⇒
0


− µ0 J0 a2
B (ρ) = θ̂

b≤ρ≤c
Z 2π
1 2π ρ
Z  Z Z   

− → − 1
J · d S = µ0 J0 πa2 − ρdρdθ = µ0 J0 πa2 − π ρ2 − b2

B(ρ)ρdθ = µ0 ⇒
0 5 0 b 5

h  2 2 i

− µ0 J0 a2 − ρ −b
5
B (ρ) = θ̂

c≤ρ

1 2π c
Z Z  Z Z   

− → − 1 2
J · d S = µ0 J0 πa2 − ρdρdθ = µ0 J0 π a2 − c − b2

B(ρ)ρdθ = µ0 ⇒
0 5 0 b 5

h  2 2 i

− µ0 J0 a2 − c −b
5
B (ρ) = θ̂

Para que el campo magnético se anule en ésta última situación, (usando que c = 3a)

(3a)2 − b2 9a2 − b2
a2 = =
5 5

∴ b = 2a

Problema 4

La única real dificultad que presenta éste problema es la forma en abordarlo. Como las corrientes tienen dos
componentes, no puede aplicarse directamente la ley de Ampere, pues es difı́cil adivinar la dirección que el campo
tomará. Por definición es difı́cil también, pues el planteo de las integrales es, al menos, complejo. Tiene solución de
ésta manera, pero es trabajo innecesario. La idea es que se trabaje para cada componente de la corriente, y con ello
se generen componentes para el campo. La totalidad de ellas, superpuestas, determinan el resultado final. Para cada
caso, por separado, si es posible usar la ley de Ampere, y entonces el problema es muy sencillo. Planteando entonces:

a≤ρ≤b


Componente según θ̂, J = αρ θ̂. Suponiendo la dirección del campo generado según k̂, por la regla de la mano
derecha, (y única dependencia en ρ dadas las simetrı́as en z y θ),
Z z0 Z z0 Z ρ
α 0 ρ
B(ρ)dz = µ0 dρ dz = µ0 αln z0 ⇒
0 0 a ρ0 a

ρ


B (ρ) = µ0 αln k̂
a

− ˆ tb. con la regla de la
Componente según k̂, J = β k̂. Suponiendo ésta vez la dirección del campo generado según θ,
mano derecha, y la misma dependencia espacial para el campo, (por los mismos argumentos anteriores), tendremos:

3
2π 2π ρ
ρ2 − a2
Z Z Z  
B(ρ)ρdθ = µ0 βρdρdθ = µ0 β 2π ⇒
0 0 a 2

ρ2 − a 2
 


B (ρ) = µ0 β θ̂

Ası́, el campo magnético total para ésta zona es:


  2
ρ − a2
 ρ 


B (ρ) = µ0 β θ̂ + αln k̂
2ρ a

b≤ρ
El razonamiento es análogo, y los cálculos, muy parecidos. El resultado final es:
  2
b − a2
   

− b
B (ρ) = µ0 β θ̂ + αln k̂
2ρ a

ρ≤a
Aquı́ el campo total es cero. Esto no puede sólo concluirse de la ley de Ampere, puesto que si una integral de lı́nea
para una curva cerrada es nula, no necesariamente el integrando es siempre nulo. Eso sı́, el vector campo magnético
debe ser nulo si se sabe que no depende de las variables de integración. Esto, pues si no fuera ası́ la integral no serı́a
cero. En éste caso puede argumentarse que, dadas las simetrı́as, en el mejor caso puede haber dependencia de ρ, pero
nada más. Adicionalmente, de la expresión general por definición, dada la ausencia de densidad de corrientes y usando
las simetrı́as del problema, (que permiten separar el desarrollo espacialmente), se concluye similarmente.

4
Enunciado Aux. No8 FI2A2
Prof. Auxiliar: Felipe L. Benavides
Fecha: Lunes 6 de Octubre de 2008

Problema 1

Medios Magnéticos

a) Un solenoide de radio 2[cm] y largo 1[m] está enrollado con un alambre fino a razón de 60 vueltas por
centı́metro, transportando una corriente de 4[A]. El interior del solenoide está lleno de un material paramagnético
de susceptibilidad magnética 2.9 · 10−4 .



i. ¿Cuál es la magnitud de H dentro de la substancia?



ii. ¿Cuál es la magnetización M dentro de la substancia?



iii. ¿Cuál es la magnitud de la inducción magnética B , en el interior de la substancia? ¿Cuál serı́a la magnitud


de B si el solenoide estuviera vacı́o?



iv. ¿Cuál serı́a la magnitud de B si en el interior del solenoide se sustituyera el material paramagnético por
hierro dulce, de permeabilidad magnética relativa µr = 500?

b) Considere
 otro solenoide, que puede suponerse muy largo, ésta vez con núcleo de hierro dulce, que posee
2000 vueltas
m , y que transporta una corriente de 20 [mA]. Con ésta corriente, la permeabilidad relativa del núcleo
de hierro es 1200. Suponga que se extrae el núcleo de hierro. ¿Cuánta corriente es necesario hacer circular por el
solenoide para producir el mismo campo dentro del sistema, que en la situación inicial?

c) Una esfera de material magnético de radio R se coloca en el origen de coordenadas. La magnetización de la


esfera es:



M = ax2 + b î


En que a y b son constantes conocidas.

i. Determine las corrientes de magnetización.

ii. Ppto: Calcule el potencial magnético vector y el campo magnético, en una posición muy lejana a la esfera
(suponiendo que el radio R no es comparable con la distancia a la que se observa).

1
Problema 2

Inductancias Mutuas

a) Calcule la inductancia mutua M del circuito de la figura si la bobina 2 posee N espiras, y la 1, n espiras por
unidad de largo.

b) Calcule el coeficiente de acomplamiento k del circuito.

Figura 1

Problema 3

Solenoide en Precisión

a) Calcule el campo magnético sobre el eje de un solenoide de largo L y radio R que es recorrido por una corriente
i, que le da N vueltas.

b) Un cilindro macizo de largo L y radio R está cargado uniformemente con densidad de carga ρ. Se hace rotar
en torno a su eje con velocidad angular constante w. Calcule el campo magnético sobre su eje.

2
Resolución Auxiliar No 8
Prof. Aux.: Felipe L. Benavides
Fecha: Lunes 6 de Octubre de 2008

Problema 1

a)

i. Si se desprecian efector de borde, (i.e., se aproxima el campo dentro de la bobina por el de una muy


larga), el campo magnético H es, (por Ley de ampére, usando que el campo magnético fuera de la bobina es
nulo),


H = nI k̂
Donde k̂ es un vector unitario a lo largo del eje del solenoide, y n es el número de vueltas del enrollado,
por unidad de longitud. La magnitud del campo es finalmente,
  

− 4 A
H = 2.4 · 10 k̂
m



ii. La magnetización M está dada por:
  

→ →
− A
M = χm H = χm nI k̂ = 6.96 k̂
m


− →

iii. La magnitud de B es µr µ0 veces la magnitud de H . Aquı́, µ0 = 4π · 10−7 y µr = 1 + χm = 1.00029.
Utilizando el resultado de i., queda,



| B |= 3.0168 · 10−2 [T ].

Si el solenoide estuviera vacı́o,


− →

| B |= µ0 | H |= 3.0159 · 10−2 [T ]

iv. Si se sustituyese según el enunciado,




| B |= 4π · 10−7 · 500 · 2.4 · 104 w 15 [T ].



b) Igual que en a), el campo H al interior del solenoide queda

1


H = N I k̂
Con I la corriente que circula por el solenoide, y N, el número de vueltas por unidad de longitud del
enrollado. Como en el interior se ha puesto un núcleo de hierro dulce, la magnetización es

→ →

M = χm H
(Como la curva de histéresis del hierro dulce es muy delgada, se suele asumir un comportamiento lineal entre

→ → −
M y H para éste tipo de material ferromagnético, al menos en un tramo suficientemente grande). Finalmente

− −
→ → − →
−  
B = µ0 M + H = µ0 µr H = µ0 µr N I k̂ = µ0 1200 · 2000 · 20 · 10−3 ∼ 0.06 [T ] k̂

Para un solenoide vacı́o,




B = µ0 N I k̂
Para producir 0.06 [T ], se necesita entonces una corriente de
0.06
I= ∼ 23.9 [A]
µ0 N

c)

i. Éste es un caso muy sencillo. En general se tiene que:


(→− −

J = ∇ × M (1)

− −→
K = M × n̂ (2)

Donde n̂ es la normal a la superficie. Vemos que la expresión (1) resulta nula, en cambio la (2),

− −→  
K = M × n̂ = aR2 sen2 (θ)cos2 (φ) + b sen(θ)k̂ − cos(θ)ĵ

Usando coordenadas esféricas, en que

x = Rsen(θ)cos(φ),

n̂ = r̂ y r̂ = cosθk̂ + senθ(cosφî + senφĵ).

ii. Basta plantear la integral del potencial magnético vector en términos de lo recién calculado y resolver,


suponiendo que el denominador puede salir de ella. De ahı́, calculando el rotor de A se obtendrá el campo


magnético B .

Problema 2

a) En general, la definición de la inductancia mutua plantea que:


φ12 φ21
M= =
I2 I1
donde φ12 es el flujo de campo producido por el circuito 2 sobre el 1, y φ21 , análogo pero inverso. Dado que
la bobina 2 es más ancha y corta que la 1, realizar ley de ampere con la suposición que el campo sólo apunta en
su eje de simetrı́a supone despreciar efectos de borde que son, en principio, no pequeños. (Ni necesariamente
despreciables). Por lo anterior, trabajaremos con el flujo producido por el solenoide 1, dentro de la 2, i.e., con
el lado derecho de la expresión. Ası́,

− →

Z
N N
M= B 1 · d S 2 = µ0 nI1 πa2 = N a2 nπµ0
I1 Bobina 2 I1

2
b) El coeficiente de acomplamiento es,
M
k=√
L1 L2
De la definición para cada autoinductancia,

φ1 nDB1 πa2
L1 = = = µ0 πa2 n2 D
I1 I1

µ0 Nd I2 πb2 N

N 2 πb2
L2 = = µ0
I2 d
Finalmente, (después de un depeje algebraico),
r
a d
k=
b D
Vemos que, si a = b y d = D, i.e., cuando ambas bobinas se convierten en una sola, se cumple que k = 1,
i.e., el acomplamiento es completo, lo que es coherente con el sistema. Si bien se utilizó aquı́ la ley de Ampére
para obtener el campo dado por cada bobina, debe recalcarse que ésto es una aproximación, en especial para
el caso de la bobina 2, y por lo tanto el coeficiente de acomplamiento aquı́ obtenido es una aproximación
solamente. Dependiendo de los tamaños y las proporciones del sistema su validez puede aumentar o disminuir.

Problema 3

a) Para el cálculo en ésta parte no es útil la ley de Ampere pues la trayectoria cerrada utiliza trozos en
los que no conocemos el valor del campo. Nuevamente la integral por definición es bastante compleja, y es
preferible trabajar con el vector potencial vectorial y luego tomar rotor. Ésto último queda propuesto, puesto
que usaremos en la solución la misma técnica que en el Problema 3 antes planteado. Calcularemos entonces el
campo magnético sobre el eje de un anillo circular de radio R, y con él, dividiremos el solenoide en espiras y


sumaremos todos los aportes. Nota: En rigor, el vector H se denota intensidad de campo magnético, y el

− →

vector B , sólo campo magnético o inducción magnética. En éste caso, se calculará H .
Según la figura 1,

Figura 1


− −

− i d l ×→r
dH =
4π r3
Con:


r = z k̂ − Rcos(φ)î − Rsen(φ)ĵ



d l = (−Rsenφî + Rcosφĵ)dφ

 23
r 3 = z 2 + R2

3
Entonces,


− i zRcosφî + zRsenφĵ + R2 k̂ →
− →
− iR2 k̂
Z
d H (z) = 3 dφ ⇒ H (z) = d H (z) = 3
4π (z 2 + R2 ) 2 0 2 (z 2 + R2 ) 2
Ésto resulta ser el vector intensidad magnética sobre el eje z, de un anillo circular de radio R, centrado en
el origen y a altura z = 0. Dividamos ahora el solenoide en espiras de espesor dz 0 . Sea m el número de vueltas
0 0
por unidad de longitud, m = N L . Entonces, en la espira de espesor dz habrá mdz vueltas, y la corriente total
por ella será

i0 = imdz 0
El campo magnético entonces, (considerando que las espiras están en z’), queda:


− imdz 0 R2 k̂
d H (z) = 3
2 ((z − z 0 )2 + R2 ) 2
El campo magnético total, (producido por todo el solenoide), a la altura z, resulta:
" #L
L L

− →
− N iR2 k̂ dz 0 N iR2 k̂ −(z − z 0 )
Z Z
H (z) = d H (z) = 3 = p
z 0 =0 2L ((z − z 0 )2 + R2 ) 2
0 2L R2 (z − z 0 )2 + R2 0
" #

− N ik̂ z z−L
∴ H (z) = √ −p
2L z 2 + R2 (z − L)2 + R2


b) Ésta parte es bastante mecánica, aunque no por eso sencilla. Buscamos nuevamente H , y para obten-
erlo planteamos por definición y resolvemos. La ley de Ampere tampoco sirve aquı́ pues nuevamente, dada
la dirección del campo, el camino necesario involucra campos de los que se desconoce su valor. Por ello,
parametrizando, tendremos:
Z →
− →

− 1 J (−
r 0 ) × (→
−r −→−r 0) 0
H (z) = →
− →
− 0 3
dV
4π k r − r k
Con:


− →
− −0
r = z k̂, →

r 0 = r0 ρ̂ + z 0 k̂ = r0 cos(θ)î + r0 sen(θ)ĵ + z 0 k̂, J (→
r ) = ρr0 wθ̂ = ρr0 w(−sen(θ)î + cos(θ)ĵ)

Ası́,
R L 2π

− (z − z 0 )r0 cos(θ)î + (z − z 0 )r0 sen(θ)ĵ + r02 k̂
Z Z Z
ρw
H (z) = 3 dθdz 0 r0 dr0
4π 0 0 0 [(z − z 0 )2 + r02 ] 2
R L
r03 dz 0 dr0
Z Z
ρw
= k̂ 3
2 0 0 ((z − z 0 )2 + r02 ) 2
Después de algo de trabajo con ésta integral, (primero integrando respecto de dz’ y luego en dr’), se concluye
que:
 

− ρw p 2
q
2 2 2 2 2
H (z) = z z + R − z + (z − L) − (z − L) (z − L) + R
2

4
Enunciado Auxiliar No9 FI2A2
Prof. Auxiliar: Felipe L. Benavides
Fecha: Miércoles 29 de Octubre de 2008

Problema 1

Se tiene un circuito conductor rectangular, con aristas a(t) = a0 sen(wt) y b, cruzado por un campo magnético


uniforme y oscilante: B = B0 cos(wt)k̂ que sale de la figura. El circuito está interrumpido por un condensador de
capacidad C. Desprecie efectos autoinductivos.

a) Determine cómo varı́a la carga en la cara 1 del condensador. Suponga que el conductor rectangular tiene
resistencia despreciable. En particular diga cuanto vale la carga en t = 0. ¿Tiene la corriente del circuito la misma
frecuencia w que a(t) y que B(t)?

b) Calcule la carga en el condensador, Q(t), en el caso en que el rectángulo tiene resistencia R, y suponiendo
que la corriente inicial, I(0), es la misma que en el caso anterior.

Figura No 1

Prolema 2

Una varilla conductora, con resistencia R, se puede deslizar por una horquilla de resistencia despreciable, fija en el
espacio, como se indica en la figura. El plano de la horquilla es vertical, y lo atraviesa un campo magnético perpen-


dicular, uniforme, y constante, B . Hay contacto eléctrico entre la varilla y la horquilla, de modo que constituyen
un circuito eléctrico cerrado. Si la varilla tiene masa m, calcule la velocidad con que cae, dada la existencia de la
gravedad, si parte desde el reposo. Desprecie efectos de roce, y efectos autoinductivos.

Figura No 2

1
Problema 3

Una espira cuadrada de lado a y masa m puede girar libremente en torno a uno de sus lados, que se elige
como eje z. La espira tiene una resistencia R. En el semiespacio 0 ≤ y existe un campo magnético uniforme,



B = B0 î
5ma2
y en el resto del espacio el campo es nulo. Asuma que el momento de inercia de la espira es In = 12

a) Suponga que en t=0 la espira tiene velocidad angular → −


w = w0 k̂, y que se encuentra en el plano y = 0.
Despreciando efectos de autoinducción, encuentre la velocidad angular de la espira, una vez que ésta ha entrado en
la región y > 0, como función del ángulo θ entre la espira y el eje î : w(θ). En particular calcule la diferencia de
velocidad angular que la espira sufre al salir del semiespacio 0 ≤ y.

b) Calcule la energı́a disipada en la resistencia durante el tiempo que la espira permaneció en la región donde
hay campo magnético.

Figura No 3

2
Resolución Aux. No9
Prof. Auxiliar: Felipe L. Benavides
Fecha: Miércoles 29 de Octubre de 2008

Problema 1

a) Para obtener la carga en el condensador, en las distintas caras, usamos la conocida relación C = VQ . Dado que
C es dato, basta encontrar el potencial que enfrenta el condensador, y se tendrá lo pedido. Notamos que, dado
que el campo magnético es variable en el tiempo, y el área del circuito que lo enfrenta también, existe inducción
de una fem, pues el flujo que atraviesa el área que encierra el circuito es, en efecto, variable. Ası́, por inducción,


ε=−
dt
Por otro lado, el flujo por el circuito es:
I

− → −
φ= B · d S = B0 a0 bsen(wt)cos(wt)
Circuito

Como sabemos,

sen(2wt) = 2sen(wt)cos(wt)

Luego,
B0 a0 b
φ(t) = sen(2wt)
2

 
dφ(t) d B0 a 0 b
⇒ε=− =− sen(2wt) = −B0 a0 bwcos(2wt)
dt dt 2

La fem que se indujo es negativa. (Siempre es ası́!). Ésto significa que la corriente que genera dicha fem se


opondrá al campo magnético original, B = B0 cos(wt)k̂. Por ésta razón, usando la misma figura, la corriente circuital
será coherente con la regla de la mano derecha para el campo contrario, no para el original. De ahı́, que la cara
1 reciba una carga positiva, pues el condensador enfrenta potencial positivo desde la cara 1 a la 2. Si rotamos la
fuente, debemos mantener ésta convención, pues lo importante no es el dibujo, sino la dirección del flujo de corriente.
Como entra por la cara 1, es positiva para ésta cara, y negativa para la 2. Finalmente, la carga inducida en la cara
1 es C · V, y en la 2, −C · V, donde V es la tensión inducida en módulo, y no hay ambiguedades, puesto que la
dirección del campo magnético original está bien definida.

1
Figura No 1.

Calculando,

Q1 = +CB0 a0 bwcos(2wt)

Para obtener la carga al instante inicial, basta evaluar en t = 0 la expresión anterior. Se observa que la frecuencia
de la carga, y por ello, de la corriente circuital, es el doble de la variación del campo. Esto tiene sentido, pues
la variación de flujo está directamente relacionada con tanto el cambio en el valor del campo magnético, como el
cambio en el área circuital por la alteración de a, en a(t).

b) Planteando la ley de voltajes de kirchoff, se obtiene, (según la figura 2),

Figura No 2.

Q
V (t) = F em(t) = VC + VR = + RI
C

Q dQ
⇒ F em(t) = +R
C dt
Aquı́ aparece una edo simple a resolver, pero larga. Se obtendrá finalmente que

Q(t) = Q(t)Homogenea + Q(t)P articular

Donde la parte homogénea viene dada por:


t
Q(t)Homogenea = Ae− RC

Resolver queda propuesto. La condición inicial es la misma que se obtuvo en a), y la solución particular viene
de proponer una solución de forma similar a la del lado derecho y calcular.

2
Problema 2

Para calcular la velocidad con la que cae la varilla, debemos primero hacer un análisis de fuerzas. Con él, y la
ecuación de Newton, se resuelve todo el problema. La idea es que el campo magnético genera un flujo variable
sobre el circuito, pues la varilla que lo cierra, se desplaza. La corriente por el circuito, y la interacción entre ella
y el campo magnético original, generarán la fuerza que se opone a su caı́da. Planteando las fuerzas obtenemos:



F Gravedad = mg k̂

Z

− →
− → −
F M agnetica = I dl × B
V arilla


− ˆ pues la corriente avanza en
Usando el mismo análisis del problema anterior, la dirección del paso dl es según i,
ésta dirección, dado que el campo que genera, (por la regla de la mano derecha), se opone al original sólo en ésta
disposición. De ésta forma, integrando sobre la varilla,
Z


F M agnetica = I dxî × (−B ĵ) = −IlB k̂.
V arilla

Luego,


F = (mg − ILB)k̂

La corriente eso sı́, es desconocida. Por ello, usamos la ecuación que entrega el efecto de inducción,
dφ d
ε=− = − ((−B)z(t)l)
dt dt
Como la vara tiene resistencia R,

dz(t) BL d
ε = RI = Bl ⇒I= (z(t))
dt R dt

d2 l2 B 2 d
⇒m (z(t)) = mg − (z(t))
dt2 R dt
Finalmente, (resolviendo el polinomio caracterı́stico, (y con él las partes homogénea y particular), sumando y
derivando),
d mgR −(BL)2
v(t) = (z(t)) = 2 + Ae mR t
dt (BL)
La condición inicial es de reposo en t = 0, por lo que,
 
d mgR (BL)2
− mR t
(z(t)) = v(t) = 2 1 − e
dt (BL)

Problema 3

a) Necesitamos una ecuación para w(θ), de tal forma que evaluando obtengamos la diferencia. Para encontrarla, us-
2
aremos torque, pues de hecho, tenemos el momento de inercia In = 5ma
12 . Como,
X→

T = In →

α

Con α la aceleración angular, y usando la aproximación de dipolo magnético, tendremos,


− →
− →

m × B 0 = T = In →

α

3

− →

Necesitamos entonces → −m. Pero →

m = I S , donde I es la corriente que atraviesa la espira, y S , el vector de área
orientada, i.e., el área orientada según la normal a la superficie. La corriente es desconocida, por lo que usamos
inducción. Tendremos que,
Z
φ(θ(t)) = B0 î · dρdz θ̂ = −B0 a2 sen(θ)


⇒ε=− (θ(t)) = B0 a2 cos(θ)θ̇
dt

ε B0 a2 cos(θ)θ̇
⇒I= =
R R


Como S = a2 θ̂


− B 2 a4
⇒ In →

α =→

m × B 0 = 0 cos2 (θ)θ̇(−k̂)
R

B 2 a4
 
1 + cos(2θ)
⇒ In θ̈ = − 0 θ̇
R 2

Resolviendo la ecuación anterior, y planteando la diferencia entre θ = 0 y θ = π,



B02 a4 6π B0 a2
4w = − π=
2RIn 5 mR

b) Ésta parte puede resolverse de dos formas, en forma simple. La primera asume el cálculo de la potencia disipada
en la resistencia por definición, usando adecuadamente las expresiones para la potencia en circuitos sencillos como
éste. Otra forma, ciertamente más elegante, está relacionada con darse cuenta que la pérdida en energı́a cinética
corresponde a la pérdida de energı́a del sistema de la espira en movimiento, y esa pérdida puede cuantificarse por
el teorema de trabajo y energı́a como
1 1
4U = In wi2 − In wf2
2 2
Es interesante notar que la disipación térmica de energı́a por efecto Joule, corresponde, en condiciones ideales,
a la pérdida de velocidad angular de la espira. Planteando los valores obtenidos para las velocidades angulares, y
el momento de inercia de la espira, se obtiene el valor pedido.

4
Enunciado Aux. No10 FI2A2
Prof. Auxiliar: Felipe L. Benavides
Fecha: Lunes 3 de Noviembre de 2008

Energı́a Magnética y Fuerzas Asociadas - Electroimán, Resorte comprimido


Problema 1

En la figura de más abajo se muestra un toroide delgado de sección circular A, que posee un enrollado de N
vueltas con una corriente I0 . El toroide se compone de dos mitades con permeabilidades magnéticas µ1 y µ2
respectivamente. Dichas mitades se encuentran separadas una pequeña distancia h. (h << a, b). Suponiendo que
el alambre conductor de la bobina tiene una resistencia despreciable, se pide estimar, (lo más preciso posible),

a) Energı́a almacenada en el sistema, en régimen permanente.

b) Fuerza sobre la parte derecha del entrehierro, asumiendo que la izquierda está fija.

Figura 1.

autoinducción e inductancias mutuas, el transformador


Problema 2

Considere un sistema formado por dos bobinas de N1 y N2 vueltas enrolladas en un núcleo de fierro toroidal de
permitividad magnética µ, según se muestra en la figura. El circuito 1 (de la izquierda) es alimentado por una
fuente sinusoidal, mientras que el circuito 2 se encuentra cortocircuitado.

Figura 2.

1
Suponiendo que los circuitos 1 y 2 tienen resistencias R1 y R2 respectivamente, se pide:

a) Calcular el valor de las corrientes I1 (t) e I2 (t) cuando ha pasado mucho tiempo desde que se conectó la fuente
de voltaje V (t).

b) Suponga ahora que cuando por el circuito 1 se encuentra circulando la corriente máxima se produce un
cortocircuito, de modo que los puntos A y B quedan unidos entre si en forma instantánea (puede suponerse que
mediante un conductor de resistencia nula). En estas condiciones se pide determinar las corrientes I1 e I2 en función
del tiempo. ¿Qué ocurre cuando ha pasado mucho tiempo (t → ∞)?

2
Resolución Aux. No10
Prof. Aux.: Felipe L. Benavides
Fecha: Lunes 3 de Noviembre de 2008

Problema 1

a) Para encontrar la energı́a almacenada en régimen permanente, encontraremos el valor de la inductancia del
sistema. Basta ésto pues, dado que la corriente I0 es un dato, los efectos autoinductivos, si no son despreciables,
ya están incluı́dos allı́. Queremos usar,
1 2
E= LI
2 0
Necesitamos L. Para obtenerla, planteamos por definición,
φ
L=
I0
Necesitamos el flujo de campo magnético por el sistema, y para ello, planteamos la ley de ampére. Aproximaremos
que h es suficientemente pequeño como para que el sistema de referencia de la primera mitad del toroide sea válido
para la segunda, y cuando desarrollemos, se encontrará que la inductancia depende de la posición, i.e., no es un
valor fijo. Ésto no permitirı́a que utilizáramos la expresión anterior, pero usaremos que el radio ρ del que depende
el valor de L es fijo; el pto. medio del toroide. Ası́,
I
− →
→ −
H · dl = N I0 = H1 (ρ)ρπ + H2 (ρ)h + H3 (ρ)ρπ + H4 (ρ)h

Usando que

B1n = B2n = B3n = B4n = B

B(ρ)ρπ B(ρ)h B(ρ)ρπ B(ρ)h


⇒ + + + = N I0
µ1 µ0 µ2 µ0


− N I0
B (ρ) ' 2h µ1 +µ2 a+b
θ̂
µ0 + µ1 µ2 π( 2 )

Asi,
2
N 2 (a−b)
4 π
L= 2h µ1 +µ2
µ0 + 2µ1 µ2 π(a + b)
Finalmente,
2
1 N 2 (a−b) π
E= 2h
4
µ1 +µ2 I02
2 µ0 + 2µ1 µ2 π(a + b)

b) La fuerza entre las mitades de toroide es el opuesto a la derivada respecto de la posición, i.e., h, de la energı́a
magnética, recién calculada. Con ello,
2

− ∂E N 2 I02 (a−b)
4 π
k F k=k − k= h i
∂h µ0 µ2h0 + µ2µ 1 +µ2
π(a + b)
1 µ2

1
Problema 2

a) Para calcular los valores de las corrientes en ambos circuitos en régimen permanente, debemos resolver las
ecuaciones de interacción del sistema y de ellas imponer que el tiempo sea muy grande. La idea es usar las
leyes de kirchoff en ambos circuitos considerando el efecto de inductancia mutua que se produce, y la autoin-
ductancia. (La inductancia mutua estará dada por la corriente del circuito opuesto, y la autoinductancia, por la
propia. Por ello se plantea el campo magnético con ambas corrientes). Para plantear dichas leyes, debemos ini-
cialmente obtener el valor del campo magnético dentro del toroide. Por ley de Ampére, debemos calcular el valor
del campo generado por cada corriente, aproximándolo en todo el toroide por el que hay en el pto. medio de él.
I
− →
→ −
H · dl = ILibre

Z π  
a+b
H(ρ)ρdθ = −N1 I1 (t) + N2 I2 (t) ρ=
−π 2


− (N2 I2 − N1 I1 )
H = θ̂
π(a + b)
El sistema de referencia es cilı́ndricas, con k̂ saliendo de la figura en la hoja. Ésto es arbitrario, y perfectamente
puede suponerse lo contrario y obtener resultados equivalentes. Por otro lado, las direcciones de las corrientes en
cada bobinado también son arbitrarias, por lo que puede suponerse cualquier cosa, y, de ser coherente, se llegará
a resultados correctos. Usando las direcciones de la figura del enunciado, i.e., con la primera corriente entrando
al sistema, saliendo desde la fuente, y con la corriente en el segundo circuito bajando por el bobinado, la regla de
la mano derecha en cada circuito dice que el flujo de campo generado por el primer circuito apuntará en −θ̂, y el
segundo, θ̂. Planteando las leyes de kirchoff en ambos circuitos, (no asumimos nada respecto de ellas),
V (t) = V0 cos(wt) + εInducida1 = R1 I1

εInducida2 = R2 I2
Las f.e.m. inducidas no son iguales, puesto que el número de vueltas en cada caso no es el mismo. De hecho,
ni siquiera tienen el mismo signo! El cálculo de los flujos en cada circuito, (inducidos respecto de su opuesto y sı́
mismos), es donde debemos tener más cuidado. Para el primer circuito, la normal apuntará según −θ̂ en nuestro
sistema de referencia. (La regla de la mano derecha para la corriente arbitraria arroja éste resultado). Para el
segundo circuito, es al revés, y la normal apunta según θ̂. Usamos las direcciones arbitrarias pues el signo menos
de la inducción ajusta después los resultados. Los flujos finalmente son, (dados los dS vectoriales),
(N2 I2 − N1 I1 )
Z
− −
→ →
φ1 = N1 B · dS 1 = −N1 µ S
π(a + b)

(N2 I2 − N1 I1 )
Z
− −
→ →
φ2 = N2 B · dS 2 = N2 µ S
π(a + b)
2
donde S = b−a 2 π
La inducción queda,

ε=−
dt

N2 I˙2 − N1 I˙1
εInducida1 = N1 µ S
π(a + b)

N2 I˙2 − N1 I˙1
εInducida2 = −N2 µ S
π(a + b)
Podemos comprobar que hemos sido coherentes, puesto que de las expresiones anteriores pueden perfectamente
extraerse los términos de inductancia mutua y autoinductancia. En cada caso, las inductancias mutuas se suman,
y las propias, se oponen al fenómeno total de tensión en el circuito. (En varios textos se encuentran éstas mismas
ecuaciones, pero con el signo cambiado. Ésto se debe a que se asume el signo opuesto dentro de los εInducccion ).

2
Finalmente, las leyes de kirchoff quedan,
 
V0 cos(wt) + C1 N2 I˙2 − N1 I˙1 = R1 I1

 
−C2 N2 I˙2 − N1 I˙1 = R2 I2

N1 µS N2 µS
usando que C1 = π(a+b) , C2 = π(a+b)
La fı́sica del problema termina aquı́. En adelante, resolver el sistema de ecuaciones permite resolver los com-
portamientos transientes. Otra forma de deducir éstas ecuaciones, eso sı́, mucho menos trabajosa, es trabajar
directamente con los términos de autoinductancia e inductancias mutuas.
En general,
Nj φjk
Ljk =
ik
donde el subı́ndice j hace referencia al lugar donde se mide el efecto, y el subı́ndice k, a quien lo genera. En
general, los flujos con subı́ndices aluden a los efectos generados por una parte del circuito, cuando la otra no
existe, o no tiene corriente. La superposición de los efectos permite resolver en forma más sencilla. Por ejemplo, la
inductancia L11 se calcula con el flujo que atraviesa el bobinado 1, generado por sı́ mismo, cuando el bobinado 2 no
tiene corriente alguna. La inductancia L22 se calcula con el flujo que atraviesa el bobinado 2, también generado por
sı́ mismo, cuando el bobinado 1 no tiene corriente. Finalmente, la inductancia L12 , (equivalente a L21 ), se calcula
con el flujo que atraviesa el bobinado 1, (sin corriente), dado el campo generado por el bobinado 2. Entonces,

N1 φ11 N12 µS
L11 = =
i1 π(a + b)

N2 φ22 N22 µS
L22 = =
i2 π(a + b)

N1 N2 µS
L12 =
π(a + b)

Usando ahora que,


d
(V (t)) = [R] (i) + [L] (i)
dt
sólo basta tener cuidado con los signos. Si se define que ambas corrientes entran al sistema, (ambas bajan por los
bobinados), para ser coherentes con el dibujo y los flujos generados, debemos usar que la autoinducción es positiva,
y la inducción mutua negativa, pues los flujos mutuos son opuestos. Ası́, se obtendrá que,
d d
V0 cos(wt) = R1 I1 + L11 i1 − L12 i2
dt dt

d d
0 = R2 I2 + L22 i2 − L12 i1
dt dt
que es exactamente lo mismo que obtuvimos al principio. Una forma de resolver éste sistema es despejar la
diferencia de las derivadas de las corrientes en la segunda ecuación, introducir ésto en la primera, y derivar dicha
ecuación resultante. Ésto entrega la siguiente EDO,
R1 R2 C2 wV0 N1
I˙2 + I2 = − sen(wt)
C2 R1 N2 + C1 R2 N1 C1 R2 N1 + C2 R1 N2

∴ I2 (t) = Ae−λt + Bcos(wt + φ)

3
Las constantes B y φ, se despejan de imponer que la solución es ésta, introducirla en la EDO y luego abrir los
cos(a+b) y sen(a+b) con la fórmula de la suma de ángulos. De ahı́, se tendrá que K1 sen(wt) + K2 cos(wt) = 0 con
los K constantes conocidas. Como las funciones sen y cos son ortogonales, las constantes deben ser nulas, para que
la ecuación se cumpla. Ésto despeja φ y B. La constante A se despeja de la condición inicial, que es que la corriente
al inicio en el circuito 2 es nula, y λ es conocida, de la EDO resolviendo la parte homogénea. λ = C2 R1 NR21+C
R2
1 R2 N1
.
La corriente I1 (t) puede despejarse sin necesidad de construir una EDO para ella. Del sistema, se tiene que,

C 1 R2 I2
R1 I1 + = V0 cos(wt)
C2

V0 C1 R2  −λt 
∴ I1 (t) = cos(wt) − Ae + Bcos(wt + φ)
R1 C 2 R1
Al imponer que haya pasado mucho tiempo, las exponenciales caerán, y si la fuente se mantiene, las corrientes
en régimen permanente serán sumas de sinusoidales según las ecuaciones para I1 e I2 .

b) La situación queda definida por las siguientes ecuaciones,


 
+C1 N2 I˙2 − N1 I˙1 = R1 I1

 
−C2 N2 I˙2 − N1 I˙1 = R2 I2

De ahı́,
 
C 1 R2
I1 = −I2
C 2 R1

R1 R2
⇒ I˙2 + I2 = 0 ⇒ I2 (t) = De−λ2 t
C2 N2 R1 + C1 N1 R2

Con λ2 = C2 N2 RR11+C R2
1 N1 R2
. Se ve que la corriente en ambos circuitos se disipará rápidamente en el tiempo.
Las condiciones iniciales determinan el problema. La corriente máxima en I1 sale de, despreciando la exponencial,
derivar e igualar a cero en la expresión original, considerando la fuente. La condición inicial determina la constante
D, y de ahı́ el álgebra de las expresiones resuelve el problema en su totalidad.

4
FI2002 Electromagnetismo
Clase Auxiliar 8
Profesor Auxiliar: Sebastián Fehlandt
Fecha: 26/10/2009

P3 C2 Otoño 2008 Se tiene un sistema formado por dos cables conductores por los cuales
circulan corrientes según se muestra en la Figura 3.

j
I1

I2

i


k

Figura 3

Se pide:

a) Demostrar que la fuerza entre alambres paralelos que conducen corrientes de intensidad
ˆ
I1 e I2, ambas en la misma dirección según i , es atractiva.
b) Si los dos alambres paralelos son muy largos y están separados por una distancia a, hallar
la fuerza magnética sobre un segmento diferencial del alambre 2.
FI2002 Electromagnetismo
Clase Auxiliar 10
Profesor Auxiliar: Sebastián Fehlandt
Fecha: 16/11/2009
P1. Se tiene un sistema formado por dos bobinas de y vueltas enrolladas en un núcleo de
fierro toroidal de permitividad según se muestra en la siguiente figura. Los voltajes y
indicados corresponden a las FEM inducidas en las bobinas, por las corrientes e . Se pide:

a) Suponiendo bobinas ideales, ie sin resistencia. Encuentre una expresión para el voltaje
en función de las derivadas temporales de las corrientes e . Haga lo mismo para el
voltaje .
b) Encuentre una relación entre los voltajes y que no depende de las corrientes ni sus
derivadas, sólo de los parámetros del circuito.

P2. P1 Examen 2008. Se tiene un par de conductores circulares de área A y ancho a, entre los
cuales se dispone un medio material con conductividad , y ,
según se muestra en la siguiente figura:

Entre los conductores, usando una fuente de potencial variable, se genera un campo eléctrico de
la forma . Si , se pide:

i. Calcular la frecuencia a la cual el valor máximo del vector densidad de corriente, es


igual al valor máximo de la corriente de desplazamiento.
ii. ¿Cuánto vale ?
P3. P2 Examen 2008. Se tiene un cilindro de radio a y altura h (h>>a), el cual tiene una
conductividad g. Se pide calcular lo siguiente:

i. Calcular la resistencia del cilindro entres sus dos extremos.


ii. Se desea reducir la resistencia y para ello se rodea el cilindro con una película de cobre
de ancho t. Sabiendo que la conductividad del cobre es gc, se pide determinar la
ecuación que permite calcular el ancho t de modo de reducir la resistencia en 20%.

P4. P4 Examen 2008. Considere una región del espacio donde existe un campo magnético
y un campo eléctrico . Si en t=0, una partícula de masa m y carga q
ingresa a este espacio con velocidad en la posición (0,1,0), se pide:

i. Plantee la ecuación de movimiento de la carga (en las tres coordenadas)


ii. Determine y dibuje (aproximadamente) la trayectoria que sigue la partícula.
Guı́a de Ejercicios No 1 FI2A2
Prof. Auxiliar: Felipe L. Benavides

Problema 1

En el plano XY se tiene una densidad de carga que, expresada en coordenadas polares, se calcula según la siguiente
expresión: (con a y σ0 constantes conocidas)

a3 σ0
σ(ρ) = − 3
(ρ2 + a2 ) 2

a) Determine el campo eléctrico sobre el punto del eje Z definido por z = a.

b) El campo sobre el eje Z, con z muy grande, tiende a cero. Determine la forma asintótica, (el orden de magnitud),
con que ésto sucede.

Problema 2

Se tiene una esfera de radio R con una densidad de carga uniforme ρ0 , excepto en una región esférica de radio
b < R, totalmente contenida en la primera, que no posee carga en su interior. Probar que en dicha zona, el campo
eléctrico es uniforme.

Problema 3

Considere un conductor esférico de radio R con una oquedad en su interior, la que contiene una carga puntual q


según se muestra en la figura. Determine campo y potencial eléctrico, E (r) y V (r) para r > R. (Ver figura No 1)

Figura No 1

Problema 4



Considere una región del espacio donde se tiene un campo eléctrico constante E = E0 k̂. En éste espacio se introduce
una esfera conductora descargada de acero.

a) Dibuje aproximadamente las lı́neas de campo en las cercanı́as de la esfera, y al interior de ella.

b) ¿Hay distribución superficial de carga en la esfera?

1
c) Si la respuesta en b) es afirmativa, ¿cuánto vale el campo producido solamente por la distribución superficial
de cargas al interior de la esfera?

Problema 5

En la figura se muestra un tubo de rayos catódicos como los usados en los televisores. El tubo produce un rayo de
electrones que entran a un espacio limitado entre dos placas. Éstas placas tienen densidades superficiales de carga
dadas por +σ y −σ, lo que provoca un campo eléctrico perpendicular a ellas. A una distancia L de las placas se
encuentra una pantalla de largo 2s. Determine σ tal que los electrones no se escapen fuera de la pantalla. Considere
que el campo eléctrico es nulo fuera de la región entre las placas, que los electrones ingresan con velocidad v0 en el
eje horizontal y cero en el eje vertical, y que hay gravedad. Asuma conocidas también las constantes que aparecen
en la figura, además de la masa y carga de un electrón.

Figura No 2

Problema 6


Se tiene un campo eléctrico constante y uniforme en todo el espacio, (vacı́o), E = E0 k̂, en que k̂ es un vector
unitario según el eje OZ. Se introduce luego una esfera de material dieléctrico de constante dielétrica ε y de radio
R. Como consecuencia de ésto, el campo eléctrico se deforma. Tómese el origen de coordenadas en el centro de la
esfera. LLámese región I al interior de la esfera, y región II al exterior. Se sabe, (ésto es un dato), que la solución
para el potencial en ambas regiones se escribe, en coordenadas esféricas:
(
VI = AI rcos(θ) + BI cos(θ) r12 (r ≤ R)
V = 1
VII = AII rcos(θ) + BII cos(θ) r2 (r ≥ R)

a) Calcule el valor de las constantes AI , AII , BI y BII . Con ello, obtenga el campo eléctrico en todo el espacio.

b) Calcule la densidad superficial de carga de polarización sobre la superficie de la esfera.

2
Guı́a de Ejercicios No 2 FI2A2
Prof. Auxiliar: Felipe L. Benavides

Problema 1

Continuidad de la Corriente y Evolución Temporal de Cargas Libres

Considere un sistema formado por dos placas conductoras conectadas a una diferencia de potencial V0 . En el
espacio interior a las placas se colocan dos medios dieléctricos imperfectos. La configuración, detallada en la figura
1, corresponde a un trozo de circunferencia. Suponga que las placas tienen dimensiones tales que puede despreciar
efectos de borde.


− →

a) Calcule el campo eléctrico E y la densidad de corriente J entre las placas. Obtenga, asumiendo conocidas
las dimensiones geométricas del sistema, su resistencia total.

b) Calcule la densidad superficial de carga que aparece en la interfaz, en régimen permanente.

c) Suponiendo que la fuente se desconecta del sistema en un instante dado, determine la evolución temporal de
la densidad de carga libre en la interfaz.

Figura 1.

Problema 2

Resistencias y Campos Magnéticos en Lı́neas de Transmisión

Parte I

Considere una lı́nea de transmisión compuesta por un cilindro de radio a y altura h, (h >> a), con conductividad
g. Se pide calcular:

a) Resistencia del cilindro entre sus dos extremos.

b) Suponga que se desea reducir la resistencia total, y para ello se rodea el cilindro con una pelı́cula de cobre, de
ancho t. Usando que la conductividad del cobre es gc , determine el ancho t que permite reducir la resistencia total
del cable en un 20%.

1
Parte II.

Considere ésta vez una lı́nea de transmisión coaxial llena de un material con permeabilidad magnética no lineal, con
un conductor interno sólido de radio a y un conductor externo muy delgado, de radio interior b, según se muestra en
la figura 2. Se sabe que en el conductr interno circula una corriente I0 hacia afuera de la hoja, y vuelve en dirección
opuesta por el conductor externo. En ambos conductores la corriente se reparte en forma homogénea, y ambos se
1.6H
pueden suponer, muy largos. Si la curva de magnetización del material se puede aproximar como B = 1000+H ,
calcule: (El material es el trozo gris de la fig.)

a) Campo magnético en todo el espacio.

b) Vector magnetización en el medio material.

Figura 2.

Problema 3

Electroimán

En la figura de más abajo se muestra un toroide delgado de sección circular A, que posee un enrollado de N vueltas
con una corriente I0 . El toroide se compone de dos mitades µ1 y µ2 respectivamente. Dichas mitades se encuentran
separadas una pequeña distancia h. (h << a, b). Suponiendo que el alambre conductor de la bobina tiene una
resistencia despreciable, se pide estimar, (lo más preciso posible),

a) Inductancia del sistema.

b) Energı́a almacenada en el sistema, en régimen permanente.

c) Fuerza sobre la parte derecha del entrehierro, asumiendo que la izquierda está fija.

Figura 3.

2
Problema 4

Inductancias Mutuas

Se pretende estudiar la inductancia mutua y algunos fenómenos asociados a ella en la materia, (en forma mi-
croscópica), modelando el movimiento de los electrones con anillos conductores de corrientes I1 e I2 , para estructuras
moleculares dispares, con uno o más átomos mayores que los otros.

a) Considere dos anillos conductores cuyos planos son paralelos al plano OXY, de un sistema cartesiano de
coordenadas. Los anillos llevan corrientes iguales y en el mismo sentido, y tienen radios a, y b, con a > b. Los
centros de los anillos están separados por una distancia d, ambos sobre el eje OZ.



i. Calcule el campo magnético B en el centro del anillo más pequeño, producido sólo por la corriente que circula
por el anillo más grande.

ii. Suponiendo que el campo magnético es uniforme, (d >> a), en toda la sección del anillos más pequeño,
calcule la inductancia mutua entre los circuitos. (Un valor aproximado de ella).

b) Demuestre que la inductancia mutua se puede calcular en éste caso, sin aproximaciones, con la siguiente
expresión:
I I →
− → −
µ0 dl 1 · dl 2
M = M12 = M21 =
4π r
C1 C2

donde r es la distancia entre ambos circuitos, (en la integral, dada por la distancia entre la posición indicada

− →

por dl 1 , para el primer circuito, y dl 2 para el segundo), y C1 y C2 son los circuitos 1 y 2 respectivamente. ¿Es ésta
fórmula general? Si no lo es, ¿en qué casos no puede aplicarse?

Figura 4.

Problema 5



Cálculo de B en diversas situaciones

a) Calcule el campo magnético en el origen del sistema de referencia, según la figura de más abajo. Determine
el valor numérico de lo anterior para I = 2[A] y a = 5[cm].

b) Considere una espira formada por dos semicı́rculos coplanares concéntricos, de radios a y b, con b > a
respectivamente, unidos por dos segmentos como se indica más abajo. Suponga que por la espira circula una
corriente I. Calcule el campo magnético producido por la espira en el centro de los semicı́rculos.

3
c) Considere un disco aislante de radio a sobre el que se ha depositado una densidad de carga superficial uniforme,
σ. Si el disco rota en torno a su eje de simetrı́a con velocidad angular w, calcule el campo magnético producido por
el disco, sobre su eje.

Figura 5.

Problema 6

Cargas en Movimiento en presencia de Campo Eléctrico y Magnético


− →

Considere una región del espaio donde existe un campo magnético B = B0 k̂, y un campo eléctrico E = E0 cos(wt)k̂.
Si en t = 0 una partı́cula de masa m y carga q ingresa a éste espacio, con velocidad →

u = u0 î, en la posición (0,0,1),

a) Determine la ecuación de movimiento de la carga, en las tres coordenadas.

b) Dibuje, en forma aproximada, la trayectoria que sigue la partı́cula.

Problema 7

Antenas y Lı́neas de Campo

El alambre indicado en la figura tiene una longitud L y está recorrido por una corriente I. Calcular, en un punto
P cualquiera situado lejos del alambre (r >> l):



a) El vector inducción magnética, B .

b) A partir del resultado anterior, demuestre que las lı́neas del campo magnético en el plano (xy) son circunfer-
encias. Para ello, prosiga como sigue:


− → −
i) Demuestre que dr × B = 0 corresponde a la ecuación diferencial vectorial de las lı́neas de campo magnético.

ii) Utilice (i) para probar que las lı́neas de campo magnético en xy son efectivamente, circunferencias.

4
q
0 z z 2
 
Para la aproximación, puede basarse en que, r =r 1−2 r cos(θ) + r
y argumentar en forma razonable.

Figura 6.

5
Soluciones Guı́a de Ejercicios No 2 FI2A2
Prof. Cátedra: Simón Cassassus
Prof. Auxiliar: Felipe L. Benavides
Problema 1.


− 1 V0 →
− 1 V0
a) J (ρ) = −   θ̂; E 1 (ρ) = −   θ̂;
ρ θ1 + θ2 g1 ρ θ1 + θ2
g1 g2 g1 g2
h i
θ1 θ2

− 1 V0 g1 + g2
E 2 (ρ) = −   θ̂; R =  
g2 ρ θ1 + θ2 (z − z ) ln ρ2
g1 g2 2 1 ρ1
 
V0 ε2 ε1
b) σ(ρ) =   −
ρ gθ11 + gθ22 g2 g1

c) σ(t, ρ) = σ(0, ρ)e−λt


   
V0 ε2 ε1 g1 θ2 + g2 θ1
Con σ(0, ρ) =   − yλ=
ρ θ1
+ θ2 g2 g1 ε1 θ2 + ε2 θ1
g1 g2

Problema 2.

Parte I.
h
a) R =
gπa2
r 
g
b) t = a 1+ −1
4gc

Parte II.  →

ρ < a
 B (ρ) = µ2πa
0 I0 ρ
2 θ̂

− 1.6I0
a) a < ρ < b B (ρ) = 2000πρ+I0 θ̂
 →

ρ>b B (ρ) = 0

 −

ρ < a M (ρ) = 0h



→ 1.6I0 I0
i
b) a < ρ < b M (ρ) = µ0 (2000πρ+I 0)
− 2πρ θ̂


ρ > b −

M (ρ) = 0

Problema 3.
N 2A
a) L = h i
2h (a+b)π(µ1 +µ2 )
µ0 + 2µ1 µ2

Problema 4.

− µ0 Ia
a) i. B (d) = 3 k̂
2 (d2 + a2 ) 2
µ0 πa2 b2
ii. M = 3
2 (d2 + a2 ) 2
b) La fórmula es general en los casos en que se cumple el teorema de Stokes. Ası́, su validez se acota a las
hipótesis del teorema, respecto de las propiedades de las curvas y superficies del sistema.

1
Problema 5.  

− µ0 I 1 1
a) B (0) = − + k̂
2a π 2
Evaluando,


k B (0) k= 2.06 · 10−5 [T ]
 

− µ0 I 1 1
b) B (0) = −
4 b a
" 1
#

− µ0 σω a2 + 2z 2 − 2 k z k (a2 + z 2 ) 2
c) B (z) = 1 k̂
2 (a2 + z 2 ) 2

Problema 6.
a) Pendiente.

b)

Figura No 1.

Problema 7.

− µ0 IL xĵ − y î
a) B '
4π (x2 + y 2 + z 2 ) 23

2
Guı́a de Ejercicios No3 FI2A2
Prof. Auxiliar: Felipe L. Benavides

I. Inducción, generador ideal - torques y movimiento


Problema 1.1

El generador elemental consiste en una horquilla de separación d, por la que puede deslizar una barra móvil de
masa m y resistencia R. El circuito está inmerso en un campo magnético B perpendicular al plano de la horquilla.
Con el objeto de generar una f.e.m inducida en el circuito cerrado formado por la horquilla y la barra móvil, ésta
última se desplaza paralela a sı́ misma, con velocidad constante v.

a) ¿Cuál es la potencia que se debe entregar a la barra móvil para que ésta tenga una velocidad constante v ?

b) ¿Cuánto vale la potencia disipada en la resistencia?

c) ¿Son iguales b) y c)? ¿Qué significado tiene ésto en términos de las energı́as involucradas, y la conversión
electromecánica?

Figura 1.

Problema 1.2

Se tiene una espira rectangular conductora imperfecta de largo l , ancho h, resistencia R e inercia I (en torno al
eje z), que es libre de girar en torno a dicho eje. En cierto instante se somete a un campo magnético variable en el


tiempo de la forma B = B0 (cos(wt)î + sen(wt)ĵ). Para lo que sigue, desprecie efectos autoinductivos.

a) Encuentre la f.e.m inducida en la espira para todo ángulo α y para todo instante t debido al campo magnético
externo. (α corresponde al ángulo entre el plano de la espira y el plano XZ).

b) Encuentre la ecuación de movimiento de la espira.

c) ¿Qué sucede con la espira después de un perı́odo largo de tiempo? Avale su respuesta con cálculos explı́citos.

1
II. autoinducción e inductancias mutuas, el transformador
Problema 2.1

Considere un sistema formado por dos bobinas de N1 y N2 vueltas enrolladas en un núcleo de fierro toroidal de
permitividad magnética µ, según se muestra en la figura. El circuito 1 (de la izquierda) es alimentado por una
fuente sinusoidal, mientras que el circuito 2 se encuentra cortocircuitado.

Figura 2.

Suponiendo que los circuitos 1 y 2 tienen resistencias R1 y R2 respectivamente, se pide:

a) Calcular el valor de las corrientes I1 (t) e I2 (t) cuando ha pasado mucho tiempo desde que se conectó la fuente
de voltaje V (t).

b) Suponga ahora que cuando por el circuito 1 se encuentra circulando la corriente máxima se produce un
cortocircuito, de modo que los puntos A y B quedan unidos entre si en forma instantánea (puede suponerse que
mediante un conductor de resistencia nula). En estas condiciones se pide determinar las corrientes I1 e I2 en función
del tiempo. ¿Qué ocurre cuando ha pasado mucho tiempo (t → ∞)?

III. Circuitos LRC y evolución temporal


Problema 3.1

a) Considere un circuito RC serie, (i.e., una resistencia y un condensador conectados en serie), con condición inicial
de voltaje V0 . ¿Cuánto vale el voltaje en el condensador, en función del tiempo? Asuma C y R conocidos.

b) Considere un circuito LR serie, (i.e., una resistencia y una inductancia conectadas en serie), con condición inicial
de corriente I0 . ¿Cuánto vale el voltaje en la inductancia, en función del tiempo? Asuma L y R conocidos.

c) Considere un circuito LC serie, (i.e., una resistencia y un condensador conectados en serie), con condición inicial
de voltaje en el condensador, V0 . ¿Cuánto vale el voltaje en el condensador, en función del tiempo? ¿Y la corriente
por el circuito? Asuma L y C conocidos.

d) Usando las ideas de b) y c), resuelva la siguiente situación: Se tiene un circuito LC serie, (L1 y C1 ), con
condición inicial para el voltaje en el condensador, y con la inductancia acoplada a otro circuito RL serie, (R1 y L1
conocidos), sin circulación de corriente al instante inicial. ¿Cuánto vale el voltaje en la resistencia, si la constante
de acoplamiento entre las bobinas es M ?

IV. Energı́a Magnética y Fuerzas Asociadas - Electroimán, Resorte comprimido


Problema 4.1

En la figura de más abajo se muestra un toroide delgado de sección circular A, que posee un enrollado de N
vueltas con una corriente I0 . El toroide se compone de dos mitades con permeabilidades magnéticas µ1 y µ2
respectivamente. Dichas mitades se encuentran separadas una pequeña distancia h. (h << a, b). Suponiendo que
el alambre conductor de la bobina tiene una resistencia despreciable, se pide estimar, (lo más preciso posible),

2
a) Energı́a almacenada en el sistema, en régimen permanente.

b) Fuerza sobre la parte derecha del entrehierro, asumiendo que la izquierda está fija.

Figura 3.

Problema 4.2

Un experimento que suele utilizarse para ilustrar la fuerza magnética es el de la contracción que sufre un resorte al
hacer circular por él una corriente. Para tal efecto, considere un resorte de constante elástica k y de largo natural l0 .
Supongamos que el resorte está hecho de un material conductor, que tiene N vueltas, (espiras), y que es de sección
transversal de área A. ¿Cuánto se comprime el resorte al hacer circular una corriente I por él?

V. Ecuaciones de Maxwell
Vector de Poynting, Ondas Planas, Intensidad de Ondas, Polarizacion (problemas cortos)
Problema 5.1

Un alambre cilı́ndrico recto de conductividad g y área de sección transversal A, conduce una corriente uniforme
de intensidad I. Determine la dirección y magnitud del vector de Poynting en la superficie del alambre. Integre la
componente normal del vector de Poynting sobre la superficie del alambre, de longitud L, y compare el resultado
con el calor de Joule producido por éste segmento.

Problema 5.2

a) Calcule el campo eléctrico de la radiación electromagnética en la superficie del sol, sabiendo que la potencia
irradiada por él es de aprox. 3.8 · 1026 [W ], y que su radio es, también aprox., 7 · 108 [m].
W 
b) En un dı́a soleado, la tierra recibe aproximadamente 1300 m 2 de energı́a radiante procedente del sol.
Suponiendo que la radiación está en forma de onda plana monocromática, incidiendo normal a la superficie, calcule
la magnitud de los vectores de campo eléctrico y magnético.

Problema 5.3

Una onda electromagnética tiene una frecuencia de 1000 MHz y se propaga en el vacı́o. El campo magnético es,
(B0 conocido),


B (z, t) = B0 cos(kz − wt)î

a) Encuentre la frecuencia angular, longitud de onda y dirección de propagación de la onda.



b) Encuentre el campo eléctrico, E (z, t)

c) Encuentre el vector de Poynting y la intensidad de la onda.

3
Problema 5.4

Una fuente esférica de 40 [W ] emite radiación electromagnética isotrópicamente, de longitud de onda λ0 . A un


metro de distancia, ésta puede considerarse una onda plana.

a) Escriba el campo eléctrico de la onda en el punto P de coordenadas x = cos(30)[m], y = sen(30)[m]. Considere


que el campo eléctrico es perpendicular al plano XY.

b) Calcule la amplitud del campo eléctrico.

c) Calcule el campo magnético de la onda, magnitud y dirección.

Problema 5.5

Dos ondas electromagnéticas que se propagan en la misma dirección, tienen igual frecuencia, número de onda y
amplitud. Las ondas están circularmente polarizadas, en sentido opuesto. Demuestre que la onda resultante está
linealmente polarizada, y calcule su amplitud.

Problema 5.6

Describa completamente el estado de polarización de las siguientes ondas:



a) E = x̂E0 cos(kz − wt) − ŷE0 cos(kz − wt)



b) E = x̂E0 sen(2πz/λ − wt) − ŷE0 sen(2πz/λ − wt)



c) E = x̂E0 sen(wt − kz) + ŷE0 sen(wt − kz − π/4)



d) E = x̂E0 cos(wt − kz) + ŷE0 cos(wt − kz + π/2)

Especifique si se trata de ondas planas polarizadas en un plano, elı́pticamente polarizadas o circularmente polar-
izadas. Indique la helicidad cuando corresponda.

Problema 5.7

Tres fuentes puntuales de luz monocromática de igual intensidad están igualmente equiespaciadas sobre una lı́nea
recta. (Distancia d entre ellas). A gran distancia y paralela a las fuentes se coloca una pantalla. Calcule la
distribución de intensidad de luz sobre la pantalla a lo largo de una recta paralela a las fuentes, en el caso en que
las tres tienen misma frecuencia y fase.

VI. Propagación en medios conductores


Problema 6.1

El campo eléctrico de una onda electromagnética que se propaga en un medio conductor, (buen conductor), en la
dirección ẑ, tiene la forma:

− z z
E (z, t) = x̂E0 e− δ ei( δ −wt)

r
2
δ=
µσw

donde σ es la conductividad del medio y w es la frecuencia angular de la radiación.

4

− → −
a) Encuentre la diferencia de fase que existe entre los vectores E y B , dentro del medio conductor.

b) Calcule el promedio temporal del vector de Poynting asociado a la onda escrita al principio.

c) Usando a) y b), considere la siguiente situación: Un equipo de radar sumergido en el mar está premunido de
un emisor de radiación electromagnética de 1000 Hz y de un detector que mide la radiación reflejada por un objeto
 1 
que se interpone en el camino de la onda. Considerando que la conductividad del agua de mar es σ = 6 Ωm ,
estime la distancia máxima a que puede estar un objeto para que aún pueda ser detectado por el equipo. Suponga
que la intensidad de la radiación en el punto de emisión es I0 , y que el equipo puede detectar hasta radiación de
intensidad I0 /1000.

Problema 6.2


− z z
Demuestre que la ecuación E (z, t) = x̂E0 e− δ ei( δ −wt) satisface ser el campo eléctrico de una onda electromagnética
en un medio de buena conductividad. Para ello, escriba la ecuación de ondas electromagnéticas y desprecie el


término µε ∂ E . Puede intentar resolver directamente, o escribir el campo eléctrico dado y verificar que satisfaga
2

∂t2
la expresión.

VII. Reflexión - Refracción


Problema 7.1

Considere una lámina de vidrio de espesor d e ı́ndice de refracción n colocada en el vacı́o. Sobre ella inciden ondas
electromagnéticas en dirección normal. Suponga que las ondas incidentes se propagan en la dirección positiva del
eje z y que la lámina está contenida en el plano xy.

a) Escriba la expresión del campo eléctrico y magnético de las ondas: Al lado izquierdo de la lámina, dentro de
ella y al lado derecho. En estado estacionario, al lado izquierdo y dentro de la lámina existen ondas que viajan en
direcciones +z y −z.

b) Escriba las condiciones de borde en z = 0 y z = d, en función de los campos especificados en a).

c) Encuentre la amplitud de la onda en la zona d ≤ z. Suponga conocida sólo la amplitud del campo eléctrico
incidente.

Nota: Utilice el sistema de coordenadas de la figura de más abajo, y considere que el vector de campo eléctrico
de todas las ondas apunta perpendicular al plano de la hoja, y hacia adentro.

Figura 4.

5
FI2002-3 2009A Control 1 Prof: Simón Casassus
9 de Abril de 2009 3h Ayudantes: Marı́a Jose Santander & Pablo Castellanos

(Desarrolle sus respuestas y cuide la presentación. Sin calculadora. )

I Energı́a en condensadores.
Consideramos un condensador plano con capacidad C.
1. (1.5pt) ¿Cuál es la energı́a U almacenada en el condensador?
2. (1.5pt) ¿Cuál es la fuerza de atraccion F entre las placas? (ayuda: recuerde que el trabajo en
~ )
un desplazamiento infinitesimal es dW = F~ · ds
3. (1.5pt) Dé una expresión para C y F en el caso de placas paralelas infinitas.
4. (1.5pt) En un condensador variable (ver Fig. 1) se puede regular el area entre las placas del
condensador girando una de las placas en una ángulo θ ¿Cuál es el torque τ ejercido en el eje de
un condensador variable? (ayuda: recuerde que el trabajo del torque es dW = τ dθ, y aproxime
la capacidad a la del caso plano infinito).

II Cargas superficiales en conductores.


Considere una carga puntual en la vecindad de un conductor conectado a tierra. Calcule la dis-
tribución de cargas en la superficie del conductor, y la fuerza ejercida sobre la carga. Explique las
aproximaciones necesarias. Calcule la carga inducida en el conductor integrando la densidad de cargas
superficial.

III Campos électricos en las superficies de dos esferas conductoras.


El siguiente problema ilustra una manera de entender porque las descargas eléctricas (i.e. chispas)
se concentran en las puntas de los conductores (i.e. como en los pararayos). Consideramos dos esferas
conductoras de radios a y b, cargas Q y q, respectivamente, y conectadas por un alambre conductor.
Supondremos que las esferas estan suficientementes alejadas para considerarlas aisladas (es decir sin
sufrir la influencias de sus campos eléctricos mutuos).
1. (1.5pt) ¿Cuál es el potencial de cada esfera?
2. (2.0pt) Dé una relación entre Q y q.
3. (2.0pt) ¿Cuál es el campo eléctrico en la superficie de cada esfera?
4. (0.5pt) Un ión, tales como los producidos en las descargas eléctricas, se encuentra lejos de las
esferas y atraı́do por ellas. ¿Con qué esfera terminará chocando? Justifique.
FI2002-3 2009A Control 2 Prof: Simón Casassus
7 de Mayo de 2009 3h Ayudantes: Marı́a Jose Santander & Pablo Castellanos

(Desarrolle sus respuestas y cuide la presentación. Sin calculadora. )

Relaciones útiles:
~ ·D
∇ ~ = ρL , D ~ = ◦ E,
~ potencial dipolar: p~ ·~r/(4π◦ r3 ) Fuerza de Lorentz: F~ = q(E
~ +~v × B).
~
Coordenadas esfericas: h i
~ = ( ∂Φ , 1 ∂Φ , 1 ∂Φ ), ∇ ~ = 12 ∂r2 Ar + 1
~ ·A ∂Aθ sin θ ∂Aφ
∇Φ r r ∂θ r sin θ ∂φ r ∂r r sin θ ∂θ
+ ∂φ
,
h  ∂2Φ i
∇2 Φ = r12 ∂r

r2 ∂Φ 1 ∂ ∂Φ

∂r
+ 2
r sin θ ∂θ
sin θ ∂θ
+ ∂φ2 .

I Esferas dieléctricas.
1. (4.0pt) Una esfera dieléctrica de radio a, con suceptibilidad χE = ( − 1) se encuentra
inmersa en un campo eléctrico uniforme E ~ = E ẑ.

a) Si el campo interno a la esfera es uniforme, ¿cuál es el potencial eléctrico en el


interior?
b) Modelamos la influencia de la esfera sobre su entorno como un dipolo con momento
p = V P , en que V es el volumen de la esfera y P = χE 0 Eint es la polarización
inducida. Calcule el potencial eléctrico debido a este dipolo.
c) Exija continuidad en la superficie de la esfera para encontrar el potencial φ en todo
el espacio.
d ) Confirme por el cálculo directo que la solución anterior satisface la ecuación de
Laplace, ∇2 φ = 0, y confirme que tiene el comportamiento asintótico correcto lejos
de la esfera. Concluya.
e) Confirme que D⊥ y Ek son contı́nuos en la superficie de la esfera.

2. (2.0pt) Consideramos ahora una cavidad esferica de radio a en un dieléctrico sometido


al campo E~ = E ẑ. Repita el tratamiento anterior para encontrar el potencial eléctrico en
todo el espacio.
II Efecto Hall.
Una intensidad de corriente I pasa por una placa de cobre de sección rectangular, de largo
L, ancho h y altura a, sometida a un campo magnético B, ~ con dirección perpendicular a la
placa de cobre (ver figura).
h
a

L B
I

1. (2.0pt) Muestre que la trayectoria de un electrón libre en un campo magnético uniforme,


es un circulo de radio rL = v⊥ /ωL , ωL = eB/(2m), donde e y m son la carga y la masa del
electrón y v⊥ es la velocidad inicial del electrón. Exprese rL en función de I, suponiendo
que la placa tiene un electrón por átomo, y que la densidad de átomos de cobre es n.

2. (1.0pt) Explique qué sucede con la trayectoria de electrones en los casos lı́mites L 
rL  h, y rL  h.

3. (2.0pt) Suponga que L  rL  h. Muestre que, en estado estacionario, una medida de


la diferencia de potencial V entre las caras de la placa separadas por h permite inferir B.

4. (1.0pt) Hagá una estimación del voltaje V esperado (sin calculadora), si I = 1 A, y si el


radio de un átomo de cobre es de 1 Å, h = 0,5 cm, a = 0,1 cm, e = 1,6 10−19 C. Comente.
¿Cómo mejorarı́a el dispositivo?

III Aplicaciones de la ecuación de continuidad de carga eléctrica.


1. (2.0pt) Deduzca una ecuación de continuidad que relacione las densidades de carga y
de corriente. (ayuda: Escriba la variación temporal de carga eléctrica dQ(t)/dt en un
volumen V como el flujo de la densidad de corriente ~j a través de S, la superficie que
limita V).

2. (4.0pt) Consideramos una esfera conductora de radio R = 10 cm, inmersa en el océano


a una profundidad de 10 m. Una corriente I = 1 A escapa uniformemente de la superficie
de la esfera.

a) En estado estacionario, ¿Cuál es la densidad de corriente a una distancia r del centro


de la esfera?
b) Si la conductividad del agua salada es γ = 1 (Ohm m)−1 , ¿Cuál es la diferencia de
potencial entre la esfera y la superficie?
c) ¿Cuál es la resistencia del agua entre la esfera y la superficie?
d ) ¿Cuál es la potencia disipada en el océano?
FI2002-3 2009A Control 3 Prof: Simón Casassus
11 de Junio de 2009 3h Ayudantes: Marı́a Jose Santander & Pablo Castellanos

(Desarrolle sus respuestas y cuide la presentación. Sin calculadora. )


Relaciones útiles:
D C
~ = µ◦ H
B ~ + µ◦ M ~ 0
~
B = µµ◦ H, µ◦ = 4π 10−7 T m A−1
Magnetización en materiales lineales: M ~ ◦.
~ = χB B/µ A B
0 g
~ = µ◦ ~j(~r0 ) × (~r−~r0 )3 d3 x0
R
B 4π k~
r−~
rk
~ ×H ~ = j~l B

Energı́a potencial magnética de un dipolo:
U = −m ~
~ · B. h
Fuerza de Lorentz: F~ = q(E ~ + ~v × B).
~
e = 1,6 10 −19 C, me = 9,1 10 −31 kg, mp = 1,67 10−27 kg.
Rotor en coordenadas cilı́ndricas:
~ × A| ~ r = 1 ∂Az − ∂Aθ ~ × A| ~ θ = ∂Ar − ∂Az z
∇ r ∂θ ∂z ∇ ∂z ∂r
~ × A|
∇ ~ z = ∂Aθ + Aθ − 1 ∂Ar
∂r r r ∂θ

I Inducción al caer un cuadro por una región con campo magnético.

En la región del espacio contenida entre el plano z = 0 y z = h existe un campo magnético B ~


uniforme y horizontal. Inicialmente, en la región z < 0 se encuentra un armazón rectangular conductor
de resistencia R, rectangular, indeformable, y de masa m. El plano del armazón es perpendicular a B.~
Los lados AB y CD son horizontales y tiene largo a, los lados BC y AD son verticales y tienen largo
b (b < h) (ver Figura). La aceleración de gravedad es ~g .

1. (2.0pt) Establecer, explicando todas las etapas del razonamiento, la ecuación de movimiento
del armazón.

2. (2.0pt) Resolver la ecuación de movimiento en el caso que el armazón es soltado en t = 0, sin


velocidad inicial, cuando el lado AB coincide on el plano z = 0. Despreciar las contribuciones a
~ debidas a las corrientes inducidas.
B

3. (2.0pt) Cuando el lado DC alcanza z = h, calcular: la velocidad adquirida por el armazón, y la


energı́a Q disipada en calor dentro del alambre. Comparar Q con la pérdida de energı́a potencial
gravitacional.

4. (+1.0pt) Si existe una corriente I en el armazón, estime el campo magnético en el centro del
armazón. ¿Cuál es la forma de B ~ infinitesimalmente cerca de uno de lo alambres? Use estos
~ debidas a las corrientes
resultados para justificar que se pueden despreciar las contribuciones a B
inducidas.
II Usos de un electroimán con sacado.
Consideramos un electroimán toroidal, con un sacado que subtiende un ángulo α ¿ 1 rad desde el
centro del toro. El toro tiene un núcleo de hierro, un radio promedio de a = 10 cm, y el sacado tiene
largo 0,1 cm, y hay 200 vueltas de alambre enrolladas sobre el toro.

1. (4.0pt) Use la siguiente tabla para:


a) estimar el campo magnético dentro del sacado cuando I = 5 A.
b) Compare la potencia necesaria para mantener un campo magnético en el sacado de 0.6 T,
con la requerida para mantener un campo de 1.2 T. Comente.

H (A m−1 ) 40 80 160 240 320 480 800 1600


B (T) 0.1 0.2 0.6 0.85 1.0 1.2 1.4 1.5

2. (2.0pt) Usando el electroimán estudiamos el compartamiento de materiales no-ferromagnéticos.


Ponemos a prueba la relación teórica
½ 2
e2
¾
m 2
χB = µ ◦ n − Zr ,
3kT 6me ◦
en que: n es la densidad de número de dipolos microscópicos, m es el dipolo permanente, r◦ es
el radio promedio de un átomo, y Z es el número de carga.

a) ¿Cuál es la energı́a magnética de un grano de material con volumen V , ubicado en el sacado?


b) Comente sobre el comportamiento magnético de la materia no-ferromagnética en los lı́mites
T → 0 y T → ∞. Deduzca dos tipos de comportamientos magnéticos en función de los
valores relativos de los parámetros microscópicos.

III Corrientes de magnetización.


Estudiamos las corrientes de magnetización en el material aislante que rodea un cable conductor.
Consideramos un cilindro infinito, de radio a, orientado según un eje z, lleno de un material aislante
con permeabilidad magnética relativa µ, y con un cable de cobre en su centro. Aproximamos a que la
sección del cable conductor es nula, es decir asimilamos el alambre a una lı́nea recta.
~ de intensidad magnética, en todo el espacio.
1. (1.0pt) Dé una expresión para el campo H,
~ en todas partes.
2. (1.0pt) Escriba el campo magnético B
~ en todas partes.
3. (1.0pt) Escriba la magnetización M
~ ×M
4. (1.0pt) Calcule la densidad de corriente de magnetización, j~M = ∇ ~ , en todas partes salvo
en la lı́nea de cobre y en la superficie externa del material.
5. (1.0pt) Use el teorema de Stokes aplicado a un disco
R de radio r < a, y los resultados del Punto 3,
~ en el centro del alambre.
para calcular la corriente de magnetización IM = j~M · dS
6. (1.0pt) Repita el Punto anterior, pero con r > a, para estimar la corriente neta de magnetización
en todo el alambre. Compare con la corriente IM llevada en el centro, y concluya.
sP1: Si la densidad total de carga por unidad de largo en la superficie interior es y la densidad de carga
total en la interfaz es nula entonces el campo para siempre vale 
  
   
  
    
  
  
La densidad de carga de polarización en la interfaz tiene dos contribuciones, !#"%$ y &! "' las que valen
$ * '
! "%$ )(  $+*  ,  (    ,.- ! "'  * (  /' *  ,  (    ,   !"   

Esto implica que la densidad de carga libre en la interfaz es
  
!10  * $ *  '
   * (  $+*  ' ,    

sP2: En una configuración esférica el campo en cada zona depende tan solo de la carga total encerrada
  ( 43657368 *:9 , <;>A =6  ;@ ? 5 5  -
2 ; A =6 N ;@? 5 5 
  (CBEDGFHBJICK LMI , 
>
' '
5  
donde
(con
 ;@? O )esdebe
9 P
la carga del conductor intermedio. La integral del campo eléctrico desde
ser cero y da
hasta infinito

RTS S S
  ;>A =Q  N;  ?  *
VU =
;WA =6  ; ?
 XO  ;Y?  *Z
;


Puesto que caras enfrentadas deben tener cargas opuestas (para que el campo en el interior de los con-
* ; * '$ ;
* Z ;\[
'$ ;
ductores sea cero), la cara interior de la superficie de radio es y la exterior de ese cascarón es

'$ ;
(para que sumen ). A su vez esto significa que el tercer cascarón tiene carga en su cara interior
y carga en la cara exterior (para que sumen ). ;
Con todo lo anterior en campo eléctrico en cada una de las tres zonas no triviales es
5 S 5 S 5
A ;  5 ' - *
A ];  5 ' -
A ;  5 '
El campo para
5^3 
es nulo y por tanto el potencial es una constante y los potenciales en las tres zonas
que siguen pueden ser escritos, salvo por una constante
;A   52S = 8 $ - ; S 8 -
* A  
5 = ' ;A  
S 5

8$ 8'
El último tiene constante nula porque debe anularse en infinito. Exigiendo que el potencial es una función
continua se obtiene las constantes y y resulta:
RTS S R`S S S
 ( ^_ 5 _  ,  A ;  5 *  U -  (  _ 5 _ 8 ,  * A ; 
5 *
 U -  ( 8 _ 5 ,  A ; 
5
Se ve que se cumple continuidad en
5   y en 5 
 y que en 5   es nulo. Para 5a3  el potencial, por
continuidad, es la constante RS S
A ;   * & U

2

     
sP3: Los campos de desplazamiento son radiales y por tanto normales a la interfaz de radio , lo que
implica que $ '
y que
    
$  $  - ' '

Suponiendo que 3  3W8 ) se tiene que
la carga del cilindro de radio  es ; , la integral de en una superficie de Gauss cilı́ndrica
de radio  , (

 
 $  9 =
 '  9  ;

que arroja
$= '  ; 
Puesto que la componente tangencial del campo eléctrico es continua se cumple que

 $$   ''
lo que implica que el campo eléctrico tiene una sola expresión
   (  ;  ,  
$= ' 
que implica que la direrencia de potencial es

  (  ;  ,    
$= ' 
y de aquı́ la capacidad del condensador es 

 ( $ =  ' , 
 

3
EJERCICIO 1. FI 2002 ELECTROMAGNETISMO
Escuela de Ingeniería y Ciencias - Universidad de Chile

Prof. Luis Vargas


Prof. Auxs. Sebastián Fehlandt
Enrique Guerrero Merino
Fecha: 17 de agosto de 2009

Pregunta
Considere una esfera de carga maciza de radio R y con densidad de carga en
volumen ρ (rv ) = ρ . × r, según se muestra en la Figura 1.
0
R

Figura 1.
Se pide:

a) Calcule el trabajo necesario para llevar la carga Q desde la posición inicial


en z=z0 a la posición final B ubicada en x=a (suponga que a<R) siguiendo
la trayectoria punteada.
b) Calcular la diferencia de potencial entre el punto B y el punto A (VB-VA).
c) ¿Cuánto vale el flujo del campo eléctrico a través de una superficie
compuesta de un casquete esférico de radio 4z0 centrado en el origen?

¡ GAMMBATTE KUDASAI ! (Buena suerte en japonés)

Vous aimerez peut-être aussi